You are on page 1of 185

Dec 2012

Answers Prepared by MCA OOW Nuri KAYACAN

Dec 2012

Answers Prepared by MCA OOW Nuri KAYACAN

Dec 2012

Answers Prepared by MCA OOW Nuri KAYACAN

Dec 2012

Answers Prepared by MCA OOW Nuri KAYACAN

Dec 2012

Answers Prepared by MCA OOW Nuri KAYACAN

Dec 2012

Answers Prepared by MCA OOW Nuri KAYACAN

Dec 2012

Answers Prepared by MCA OOW Nuri KAYACAN

Dec 2012

Answers Prepared by MCA OOW Nuri KAYACAN

Dec 2012

Answers Prepared by MCA OOW Nuri KAYACAN

Dec 2012

Answers Prepared by MCA OOW Nuri KAYACAN

Dec 2012

Answers Prepared by MCA OOW Nuri KAYACAN

Dec 2012

Answers Prepared by MCA OOW Nuri KAYACAN

Dec 2012

Answers Prepared by MCA OOW Nuri KAYACAN

Dec 2012

Answers Prepared by MCA OOW Nuri KAYACAN

Dec 2012

Answers Prepared by MCA OOW Nuri KAYACAN

Dec 2012

Answers Prepared by MCA OOW Nuri KAYACAN

Dec 2012

Answers Prepared by MCA OOW Nuri KAYACAN

Dec 2012

Answers Prepared by MCA OOW Nuri KAYACAN

Dec 2012

Answers Prepared by MCA OOW Nuri KAYACAN

Dec 2012

Answers Prepared by MCA OOW Nuri KAYACAN

Dec 2012

Answers Prepared by MCA OOW Nuri KAYACAN

Dec 2012

Answers Prepared by MCA OOW Nuri KAYACAN

Dec 2012

Answers Prepared by MCA OOW Nuri KAYACAN

Dec 2012

Answers Prepared by MCA OOW Nuri KAYACAN

Dec 2012

Answers Prepared by MCA OOW Nuri KAYACAN

Dec 2012

Answers Prepared by MCA OOW Nuri KAYACAN

Dec 2012

Answers Prepared by MCA OOW Nuri KAYACAN

Dec 2012

Answers Prepared by MCA OOW Nuri KAYACAN

Dec 2012

Answers Prepared by MCA OOW Nuri KAYACAN

Dec 2012

Answers Prepared by MCA OOW Nuri KAYACAN

Dec 2012

Answers Prepared by MCA OOW Nuri KAYACAN

Dec 2012

NOTES ON
SHIP STABILITY
FOR PHASE s/CLASS
4

POST/OOW

/o-

Gla

gg,H,,,tg}J,ge

Answers Prepared by MCA OOW Nuri KAYACAN

Dec 2012

A'nswbr

rnun"k

is now displacing a uolume of wialer wlere:

Voilume of ljdter clisplaced

=_(x:

2m

hm)

The crane driver now lowers the block so that itbecomes half submerged in the dock water which has a density of 1.020 Vm3.

Mass of water displaced


wale1

Volume

Density of the dock

buoyancyforce (Bfl created by the displacedwater.

=4m3x 1.020t/nf : 4.08 t whicl represents tle upthrust of 'lhe


Mass of block = 62.72 t Unlhrust due to Bf 4.08 t Gause reading : 58.64

Tlerefore:
DOCKWATER
DENSITY
1.020 t/mr

BUOYANCYFORCE ACTING AT CENTROID oF UNDERWATER VOLUME ( 4.08 0

SA'O

. : , ::: :: ::;:;::,i,, :,:, j : Wrt'at,Ioad(mass) wil the gaue noi indicae? ,,'',',::' .

',, 1,

,,,"',

',,,,

'

WEIGHT FORCE ACTING AT CENTRE OF GRAVITY OF THE BLOCK (62.72 t)


Basic principles (MAR
Frev.72108102)

Basic principles (MAR

Prev.l2l08l02)

Answers Prepared by MCA OOW Nuri KAYACAN

Dec 2012

BASIC PRINCIPLES PryryCryLJS


The densily of any given substance is it's muss per unt volume.

The laws governinq flotation Two laws need to be considered:


J.

This can be beexpressed expressed as

as: ffi DENSITY: MASS I VoLUME VOLUME


tonnes

N ^

,r

Archimedes' principle; The law offlotation.

l. Archimede's principle
that when a body is wholly or partially immersed in a liquid, it experences an upthrust (apparent loss of mass - lermed Buoyancy force (B)) equal to tle muss of liquid

I9jlhip

st1_bjlity purposes the.u-nits commonly used ae:

Mass:

(t)

!'ou-"-. cubic metres (m3) Density: tonnes per cubic metre (t/m3)
Rearranging the above formula gives:
l

displaced

l
and anci:

VOLUME: MASS MASS

Consider a block of steel measuring 2m x 2m x 2m that has a density of 7 .84 tlm3.


DENSITY DENsITY

MASS=VOLUMExDENSITY

FESE-WATER'6r7 S,er WATER (Sil1:

Density of water in which a ship tvpically floats A ship is presumed to always float in water which lies in the following density range:

ship's side in air?

1.000

t/m3

to

Water that lies between these two extremes is termed DOCK tyATER (DW).
Basic principles (MAR Rev. l2l08/02)
I

Basic principles (MAR


Piev.12/08102)

Answers Prepared by MCA OOW Nuri KAYACAN

Dec 2012

tates that every Jloating body displaces i's own of he liqaid in which itfloals.

Law of flotation

To calculate the displacement of a box-shaped vessel


Consider the vessel shown.

hi- number of onnes of water it displaces. It is usual to consider a ship displacing salt water of density I.025 tlm3, however, fresh water values of displacement (1.000 t/m3) are
often quoted in ship's hydrostatic data.

The displacement of a ship (or any floating object) is defined as

The u,olume of displacement is the underwater volume of a ship the waterline. 1float i.e. the volume below

VOLUME OF DISPL.: LENGTH

To calculate the displacement (W) of a ship the following needs


to be known: The volume of displacement (V) The density of the water in '}t lct it floats (p) Therefore:

BREADTH

DRAUGHT

Vox:LxBxd
DISPLACEMENT : VOL. OF DISPL. x WATER DENSITY
Wro* SAO
ap e d

Since:

MASS: VOLUME

DENSITY

the mass, or displacement, of a ship is calculated by:

=(LxBxd)x
a ts at a draught of 4i2
uq

DISPLACEMENT
t.e.

VOL. OF DISPL,

WATER DENSITY

e:

I' h a'.;h

as',..

Basic principles (MAR


Rev.72108102)

Basic principles (MAR Rev. l2l08/02)

Answers Prepared by MCA OOW Nuri KAYACAN

Dec 2012

: ' w (m'ass)'',will
''Whu'Ioad
'iow.'Iowers'th'e'blo;bk

;,

dock,water?

:driie,t

Anlswil',,:;t:,,

in'th'

Th

:, ::: now ater

ter
:'8
mS

Vo

. :,,:,,
,
::

:,,,

m1

DOCKWATER
DENSITY
7.020 tlm3

Therefoie:

BUOYANCY FORCE ACTING AT CENTROID oF UNDERWATER VOLUME ( 8.16 t)

DOCKWATER
DENSITY
1.020 t/mr

WEIGHTFORCEACTING AT CENTRE OF GRAVTTY OF THE BLOCK (62.72 t)


Basic principles

(MAR

Rev. 12108/02)

Basic principles (MAR Rev. l2l08/02)

Answers Prepared by MCA OOW Nuri KAYACAN

Dec 2012

Basic principles (MAR Pev.12/08102)

lt

Answers Prepared by MCA OOW Nuri KAYACAN .6

Dec 2012

Answer

, .1 ,'
.,.,;

W"o*= (Lx Bx d) x p Wnox =(80 x I6,x 4.2)


,Wnox,=

{l!o.oJ

:,

1.025
'

To calculate the displacement of a ship Since a ship is not box-shaped, a factor known as the block coeflicient (Cu) needs to be considered.

Answer

The block coefficient

(C) of a

ship is the ratio of

], Disp .'

IunexDen;siy.:
odisii.

',

',.

iotn, auorry'
:

y4-e-7waq4 yolqmg to the volume of the

circumscrbing block.

.:.

'11400:VoL of ctspl. x
1.025
?.'LB:',.'. ,:ll.
A t'

1.025 '

..

,..,:'.

. .- . :

:.

,.

140x18x5;60

Cr= v
LxBxd
Therefore:

C-: ,0.788

REMEMBER THAT C|HAS NO.UNITS,IT


An alternative solution might be as follows:
Basic principles (MAR Rev. 12108102)

IS

..:. ....-...--::.. A nero

Wrro =(LxBxdxCr)x
Basic principles (MAR Rev. 12108102)

10

Answers Prepared by MCA OOW Nuri KAYACAN

Dec 2012

3.

(a)

Defne e term 'Block coefEcient'

o)

ship has a length at the waterline of 56.2 m and breadth of 11.6 m and floats at a draught of 3.64 m in salt water. If the vessel's displacement is 2096 tonnes calculate the block coeffcient.
(0.862)

(c) (d)

Calculate the TPCsw of this vessel given that Cw is 0.822.

(s.4e3) How much more czrgo may be loaded so that the ship sails with a draught of 3.96 m in salt water? (17s.8 t)

4.

[Jse the hvdrostatc partculars data sheet for the following examples A ship arrives in port and has a draught of 5.20 m in salt water. How much cago must be discharged so that the ship may sail with a draught of 4.60 m.

(1314 t)

5.

A ship has

(a)

an arrival draught of 4.90 m in dock water What is the arival displacement?

RD

1.006.

(b) 6.

(9674.3 t) 2174 t of cargo is loaded. What will be the final draught of this ship when it enters salt water RD 1.025. (5.800 m)

A ship displaces 10,516 t in salt water. 960 tonnes of cargo is loaded.


Calculate the final draught in salt water using: the displacement and draught scales only;

(a)

(b) 7.

(s.63s m)
(s.633 m)

the appropriate TPC value.

of 6.00 m in dock water RD 1.004. How much cargo may the ship load to ensrre that the morimum draught on sailing is 6.46 m in salt (1292.7 t)

A ship has

a draught

water.

Basic principles (MAR

Rcv

22108103

Answers Prepared by MCA OOW Nuri KAYACAN

Dec 2012

GLASGOW COLLEGE

OT

NAUTICAL STUDIES
BASIC PRIT*CIPLES

l'

{9}

Statt Arhimfe'fficiple"
Ed hgs a deui of 7.96

tb} A b-Ick of teI ta drnions qs Iovtr t/t] lm

(i)
t

Calculde the mg oftfu bloc&-

(+v.760 Th blsh i* panElly imnets+d t r dp of 36 cn n do+k ugter RD l.00t. wha load i tonnes uill regstr gD thff Graffi

il

grugr?

(iii)

fdJ.JsJ fJ The crane driver now lowers the block so that it is fully immersed, what load in tonnes will now register on the crane
gauge? (41.712 t)

A box-shaped vessel has the following particulars: length 86 m, breadth 18.2 m, depth 10 m and floats at a draught of 3.6 m in salt water (RD 1.025).

(a)

Calculate the displacement of the vessel.

(b) (c) (c) (d)

(s77s.6 t) What draught will the vessel float at if it is towed into dock water RD

(j.646 m) 300 tonnes of rock ballast is now loaded into the vessel whilst floating in e dock water. Calculate the new volume of displacement.

r.0t2?

How much more rock ballast must be loaded so that the vessel will float at it's moimum permitted draught of 5.62 m in salt water (RD r.02s)?
How much rock ballast must be loaded to sink the vessel over a sand bank as part of a coastal protection scheme?

(6003.6 m3)

(2940.7 t)

(7027 t)

Basic principles (MAR Rev 2?J08103

Answers Prepared by MCA OOW Nuri KAYACAN

l .

Dec 2012

Thg hydpmeter is used to determine the relative density of fluids, including the dock water in whcn the vesseI is floating and liquid cargo densities. It maY be nteGssiy when in pot to calculate the dock water allowance by use of the formula :

Graduated scale (1000 -7}25tg/m3)

DWA=FWA(1025-D)

Where D =- dock water densi. This will then be used to ensure that js lhg ship correctly loaded to her marks. Ships iydrometers are-usually made Lead shot of polished steel or brass, though they can also be made from glass. A bulb weighted with lead shot or mercury acts to keep the graduated stem upright. The operation of the hydrometer is based upon the Laws of Floatation, where the mass of the hydrometer is constant. Tb determine the density of dock water the following procedure should be qdopted:

25

Float Chamber

,/ ,/

Ensure the hydrometer is free from any damage li p13ctlcalre severa!samples from various locations around the vessel. ,/ laught dock water samPle that ) rece-ptacte, Jpp !!e miQ , and in the ater until it-isat th There inat tne samp]e point is c|ear fr-om any gverboa1d dischar-ge, must be sutficienf depth of liquid to ensure the hydrometer floats without interference. / F_l__oa! the hydrometer in the water (once the sample is still). .Give the instrument a slight twist to break surface tension and release any trapped air. / Take a ieading from the scale, at the water level, once the instrument has settled, allowing for any meniscus. ./ Usng the DWA formulabalculA{e the DWA.

Answers Prepared by MCA OOW Nuri KAYACAN

Dec 2012

'
)RAUGH]
m

HYDRoSTATG PARTGULARS
DISPL.

DISPL.
t

TPC
t SW

TPC
t

MCTC
t-m

MCTC
t-m

KMt
m

KB
m

LCB
foap m

LCF
foap

SW
RD 1.025
7.00

FW RD
1.000

FW RD r.000
22.51

sw
RD r.025
184.6
t

FW RD 1.000

RD

1.025

r4576 t4345

14220 13996

23.t3
23.06

80.l
78.5
177.O
t

8.34 8.35

3.64 3.58 3,53 3.48 3.43 3.38 3.33 3.28 3.22 3.17

70.03 70.08

67.35 67.46
67 -57

6.90 6.80 6.70 6.60 6.50 6.40 6.30


6.20

22.50
22.43

83.0

l4l 5
3886
t3657
13429 13201

t3771
I3548 13324 13102

22.99 22.92
22.85

181.4

836
8.37 8.38 8.39
8.41

70.t2
70.16 70.20 70.24

22.36 22.29
22.23 22.17

t79.9
178.3
r

75.5

67.68 67.79 67.90 58.00 6E.10 68.20 68.30 68.39 68.43 68.57 68.65

174.O

22.78 22.72 22.66 22.60 22.54 22.48


22.43

76.8

172.5 171.0

t2879
r2558
12437 12217

175.3 173.9 172.5

702E
10.32 70.35 70.38 70.42 70.46 70.50 70.53 10.51 70.60 10.64 70.68 70.72 70.75 70.79 70.82

t2975
12748

22.t1
22.05

69.6
r68.3
167.0

843
8.46 8.49

6.r0
6.00
5.90

t2523
12297

2t.99 2t.93 2t.87


21.82 21.71

t7l.I
69.8
r68.5
167.3

1t997

t65.7
164.4

8.s2
8.55 8.59 8.53 8.67
8.7 r

3.tl
3.06
3.01

t2073
t848

l177E

s.80
5.70 5.60 5.50 5.40 5.30 5.20 5.10 5.00

il559
tt342 t1t24
0908

22.37 22.32 22.26 22.2t


22.15

t63.2

1t625 11402

66.
t65.0
163.9 162.9

t62.t

2.95

21J2
21.66

6.0
60.0
t58.9 t57.9
156.9 155.9
54.9

290
2.85 2.80

68.?3 68.80
68.88 68.95 69.02

lt80
r0958
10737

t069
10476
10260 10045

2t.61

8.76

22.10
22.05

2t.56
21.51

6l.8
60.8
t59.8
158.8 157.9

8.8r I 2.74 I
8.86 8.92
8.98 2.69 2.63 2.58 2.53

t05t6
10296 10076 9857 9638

22.00

21.46
21.41

69.09 69.16
69.23 69.29 69.35 69.42

9830 9616 9403

2t.95
21.90 21.85 21.80
21.75

4.90 4.80 4.70 4.60 4.50 4.40 4.30 4.20

2t.36 2t.32 2t.27 2t.22


21.17 21.12 21.07 21.02 20.91 20.93

54.0

9.06

56.9

9420
9202
8985

990
8978

156.0
1542
153.3 152.4

I 155.1 |

l53. l
rs2.2

9.t3
9.22 9.30 9.40 9.49 9.60
9.71 9.83

2.48
2.43 2.38

70.86 70.90
70.93 70.96

rsr.3
150.5

8766
8554

2t.70
21.65

2.32
2.27 2.22

69.48 69.55
69.62

8768

t49.6 r48.7
147.8

71.00

8552
8336

E3M
8

2t.60 2t.55
21.50
21.45

7t.04
71.08

t33

5t.5
150.6 149.7

2.t7 2.t2
2.07
2.01

69.68 69.74
69.81

4.0
4.00
3.90 3.80

8l2l
7906

7923 7713 7505

t46.9

71.t2
71.15

46.0 t45.r
144.2

9.96

7692
7478 7265

2t.40 2t.35
2

20.88
20.83 20.78 20.72 20.67

48.7
147.8

l0.

7.8
7r.22

69.88
69.94

1296
7088 6E80 6673

10.25

.96

3.70

t.30

46.8
r45.9
144.9

r43.3
142.3
t

0.4l
r0.57 ro.76

l.9l
1.86
1.8

7t.25
71.29 71.33

70.00 70.07
70.14

360
3.50

7052
6840

21.24

2t.19

41.3

WITH TTIE VESSEL FLOATING ON EVEN TI{ESE FTYDROSTATIC PARTICULARS HAVE BEEN DEVELOPED

KEEL.

Answers Prepared by MCA OOW Nuri KAYACAN

Dec 2012

Ex.

Calculate the Dock Water Allowance (DwA) of a vesse, FWA floating in dock water with a density of 1.011.

= 280mm,

,/ DWA

= FWA(1025 - D)

25

___

80(1025

owl

101

1)t25

ss.emm

A hydrometer has a bulb with a volume of 24cm3 and a uniform stem with a cross seciona area of o.22cm2. When floating in SW (1.025i 4.6cm of the stem is
immersed. Calculate the length of stem that will be immersed when the hydrometer is floating in FW'

(7.442cm)

A hydrometer has a weighted bulb radius 2.5cm and a uniform stem with a cross secional area of 0.36cm2. When floating in FW 7cm of the stem is immersed. calculate the length of stem immersed when it is floating in sw.
(5.056cm)

Answers Prepared by MCA OOW Nuri KAYACAN

Dec 2012

Answer The muss of the additional slice of displuced water is the same us the added weight 'w'.

Since:

Mass

then:

Volume x Density
-:

AQded dinl1cement

Vol' of slice x density.

If the wPA is assumed to not significantly change between


Added displacement: (WPA(m2) x
1

cm) x density;

Added displacement: WPA(mr)

"

(m)
100

density;

Thus, the formula for TPC is given by:


=--

TPC:WPAxp
100

SAO
Calculute the TPC for ship wth a wuterplane area of 1500 m2 when it is flouting in:
(a) salt wuter;

(b) fresh wuterl

@) dock water of

RD 1.005.
2

TPC (MAR Rev. 03/01/01)

Answers Prepared by MCA OOW Nuri KAYACAN

Dec 2012

TONNES PER CENTIMETRE IMMERSION (TPC)


The TPC for any given draught is the weight which must be loaded or discharged to change the ship's mean draught by one
centimetre.

Consider the ship shown floating in salt water (RD 1.025) with a waterplane area (WPA) at the waterline as shown.

A weight 'w'
increases by
1

tonnes is loaded on deck so that the mean draught

cm.

SAO
What is the mass of the udditional 'slice' of displaced water equul to?
TPC (MAR Rev. 03/01/01)

Answers Prepared by MCA OOW Nuri KAYACAN

Dec 2012

DRAUGHT
m

DISPL.
t

DISPL.
t

MCTC
t-m

MCTC
t-m

KMt
m

KB
m

LCB
foap
m

LCF
foup m

SW RD
1.02s

FW RD
1.000

SW RD
1.025

SW RD
1J.57
1.025 184.6

FW RD 1.000
t80
I

700
690 680 670 660

r4576
14345

14220

23.13

8.34 8.35 8.36 8.37

3.64

70.03 70.08

67.35 67.46

t3996
t3771
r

B.A6
22.99 22.92 22.8s 22.78 22.72 22.66 22.6A 7? \4
22.45

r83 0 ?2,43
181.4 179.9

t78 5
177.0 175.s
174.0

358
353

l4l l5
t3886

70.12
70.16

67.57
67.68

3548

t3657
13429 13201

13324 13102 12879

ln ao

348
3.43

178 3 176 8

6s0
6.40

838 839 841


843
8.46

70.20 70.24
70 28

67.79 67.90
68 00
68.

t72.5

338
J.JJ

ll5

t7t 0
169.6
168 3 167 0

630
6.20

t2975 t2748 12s23


12297 12073
I

t2658
t2437
12211

173.9 172.5 171


1

328
322
3.17

70.32
70.35 70.38

l0

610
6.00 5.90 5.80 s.70

68.20 68.30 68.39


68.43 68.57 68.65 68.73

21.99

849
8.52

11997

22.
22.37

2t.93

169.8 168 5

16s.7
164 4 163.2

311
3.06 301 2.9s 2.90
2.85

70.42 70.46 70.50


70 53 70.57

11778
I

85s
859
863
8.67

1848

1559

21..82

167.3 166
I

t162s
11402 r1180 109s8 10737
10516

11342
11124
I

21.77

162

560 550 540


5.30 5.20 5.10

n,26
22.2t
22.15

21.12
21.66

165 0

161 0 160 0

0908

163.9 162.9 161 8

871 876
8 81

70.60 70.64
70.68

68.80
68.88

10691

2l'6
21.51

ls8

280
2.74

10476 10260
I

22.10
22.05

157 9

68.9s
69.02 69.09 69.16 69.23 69.29 69.35 69.42 69.48 69.55 69.62 69.68 69.74 69.81 69.88 69.94 70.00 70 07

160 8 159 8 158 8

t56 9
155.9

8.86 8.92

269
2.63

70.72
70.75

10296 10076 9857 9638

0045

22.00
21,95

21.16
?1.41

500
4.90

9830 9616 9403 9190


8978 8766 8554

1549
t54.0
153
1

898 906

258
2.53 2.48

70.79 70.82
70 86

2t.90 2I.E5
21.80

t57.9 1s6.9 r56 0 155


I

480 470 460


4.50

913
9.22

9420 9202
8985 8768 8552 8336

152.2

243

70.90
70.93

2'75
21.70
21.65

t51.3 t50 5
149.6

930

238
2.32 2.27 2.22 2.17

2t-17
2L..12.

154 2 153.3
152 4

940 949 960


971 983

70.96 71.00 71.04


71.08

440 430 420 410 400


390 380

83M
8133 7923

2tffi
21.55

t48.7 t47.8 t46.9


146.0

2t.02

51.5

82l
7906
7692 7478 7265

2l:50
21.45

rs0 6
149.7

2.t2
2.0'7 2.01

71.12
71.15 71.18

7713

996
10.1 I

750s
7296
7088 6880

2t.40 2t.35 2.3o


21.24

?.88
20.78

487
47.8
46.8

t45

t44 2
143 3

t0.25

1.96

71.22
71.25

370
3.60 3.50

l0
10

41
5',7

r91
1.86

7052
6840

459
24.67
144.9

1423
141 3

71.29

6673

2t.le

10.76

l8t

7t.33

70 14

THESE FIYDROSTATIC

PARTICIJLAR'PVPFpppR@I'pry9/UI7Uy

VESSEL FLOATING oN EVEN

KEEL.

Answers Prepared by MCA OOW Nuri KAYACAN

Dec 2012

Answer

TPC-ru.x
100

u) b) c)

TPC -

7fl x
100

1.025

75.375

TPC:

!fl x
100

1.000 = 75.000

TPC : 1500
100

1.005

75.075

Consideration of the TPC formula indicates that: *- -iPc increases with WPA und for a normul ship-shape the WPA wiII increase witl druught. ,< TPC increases with density. Two vulues of TPC are often quo(ed in ship's hydrostatic data, TPCswund TPCFW

TPC values for the draught range of a ship allow us to calculate how much cargolballast etc. to load or discharge to achieve a
iequired draJght.

SinkagelRise (cms)

TPC

where

represents the total weight that discharged to change the draught of the ship'
TPC (MAR Rev. 03/01/01)

'w'

is

loaded or
J

Answers Prepared by MCA OOW Nuri KAYACAN

Dec 2012

Answer Method I Initial draught 5.10

m Arr: 102g6 t Required draught 6.40 m Arr= 13201 t Curso to load - 2905 t
TPCsw: 22.00 TPCsw:22.72

Method 2 Initial draught 5.70 m Required draught 6.40 m

Meun TPCsw:22.00 * 22.72


Sinkage (cms)

22.36
7,30 m = 130 cms

2 6.40 m - 5.10

m:

Sinkuge (cms)

w TPCSW

Curgo to loud (w)

: Sinkuge

Mean TPCsw

730

22.36

2906.80 t

The answers may differ slightly because in using the mean value of TPC it is assumed that the TPC value will change
linearly between the range of draughts concerned.

SAO Had the ship been floating in fresh water (Frl/), would the
amount of crgo to load be the Sme to uchieve the required draught of 6.40 m in fresh wteF.
TPC (MAR Rev. 03/01/01)
6

Answers Prepared by MCA OOW Nuri KAYACAN

Dec 2012

SAO

shp has an initiul mean druught of 5.10 m in salt wuter and is required to complete louding with a druught of 6.40 m. Using the hydrostutic particulars culculute the amount of cargo that must be loaded.

The answer may be calculated in two ways:

Method 1 1. Read off the displacements (SW) for both the initial and required final draughts.

2. 3.

Subtract the smaller from the larger. Result equals the amount to load.

Method 2 1. Read off the TPC ,, values for both the initial and required final draughts. 2. Calculate the mean TPCsw value. 3. Calculate the required change in draught; in this case

4.

sinkage. Use the formula:

Sinkage/Rise (cms)

TPC

to find '\M', the amount to load.

TPC (MAR Rev. 03/01/01)

Answers Prepared by MCA OOW Nuri KAYACAN

Dec 2012

Answer Method 1 Initial draught 5.10 m Required draught 6.40 m

CARGO TO LOAD

Method 2

Initial druught 5.10 m

Required druught 6.40

TPCFW:27.46 TPCFW:22.77

Mean TpC_,,,= 21.46


Sinkage (cms)

22.17

_ 2I.BIs
m :1.30 m:130 cms

, 6.40 m - 5.70

Sinkage (cms)

TPCFIT

Curgo to loud (w)

: Snkage x Mean TPCFW


:130

2k

21.815:2835.95 t

The displacement for the correct density must be used in all calculations. The TPC for the density in which the ship is loading in should be used in calculations.
It is usual to calculate the amount to load on the basis of the required salt wuter draught.
TPC (MARRev.03/01/01)

"k

Answers Prepared by MCA OOW Nuri KAYACAN

Dec 2012

a rectangle having the Same length and breadth of the ship at the waterline in question.

Coefficient of fineness of the water-plane area (C*) Is defined as the ratio of the ship's water-plane areato the area

Waterline breadth
I

Waterline length

Cw:WPA

LxB

Since the ship's WPA is less in area than the rectangle formed around it , the vulue of Ca must ulways be less than 7.00.

Basic principles (MAR Rev. 03/01/01)

Answers Prepared by MCA OOW Nuri KAYACAN

Dec 2012

FORM COEFFICIENTS
Form coefficients are ratios which numerically compare the llipts underwater form to that of a regular shape (such as a
rectangle or box-shape).

They ate primarily used a the design stage, prior

to

construction, to determine factors such as resistance to forward motion hat the ship will experience during operation, this then being used to determine the ship's power requirements/engine(s) size.

Design coefficients o{primary concern are:

CoeIcient of Jineness of the wuter-plune are (Cw)

Block coefficient (C)


Midship s
co

efJicient (C n)
(Cp)

- Longitudinal prismutic cofficient


_1

Jl

Basic principles (MAR Rev.03/01/01)

Answers Prepared by MCA OOW Nuri KAYACAN

Dec 2012

Midships coefficient

any draught is the ratio of the underwater transverse area of the midships secti_on tg the product of the breadth and draught (the surrounding rectangle.

fhe midships ioefficient- (cr) of a ship at

(Cnn)

raught

Bread

CM

Underwater transverse area of mid@ Breadth x Draught

(A_)

Similarly, the vulue of Cmust always be less than 7.00.

Basic principles (MAR Rev. 03/01/01)

Answers Prepared by MCA OOW Nuri KAYACAN

Dec 2012

Block coefficient (Cr) The block coeffcient (Cr) of a ship is the ratio of the underwater volume of a ship to the volume of the
circumscribing block.

cs:

Volume of displacement

LxBxd

Therefore:

Displacement.r,, - (Lx B x d x Cr) x P

Since the ship's volume of displacement is less than the volume of displacement of the surrounding block, the vulue of C o must alwuys be less than 7.00.

Block coefficient is an important factor when the assigned ne"UoarO of a ship is being calculated (see 'Calculation and
As
si;ignment

of Freeboard' notes.)

Basic principles (MAR Rev. 03/01/01)

Answers Prepared by MCA OOW Nuri KAYACAN

Dec 2012

Longitudinal prismatic coefficient (Cr) The longitudinal prismatic coefficient (Cr) of a ship at any
dryught is the ratio of the underwater volume of the ship to the volume of the prism formed by the product of the transverse are?of the midships s_ection and the waterline length.

C, -r : Volume

of dispacem Volume of prism

cp:

Volume of displacement of shi Waterline length x Area of midship section (Am)

This coefficient gives an indication of how much the ship's form changes at the ends. SimilarlY, the vulue of Cp must ulwaYs be
Iess thun 7.00.
Basic principles (MAR Rev.03/01/01)
5

Answers Prepared by MCA OOW Nuri KAYACAN

Dec 2012

Dock water allowance (DWA) T-he bock Water Allowance (DWA) of a ship is the number of millimetres by which the mean draught changes when a ship
passes from salt water to dock water, or vice-versa, lyhen the ship is loaded to the summer displacement.

The DWA is a fraction of the FWA and is found by the formula:


(.'\

DWA (mm): FWA x


25

Note The densities are multiplied by 1000 to simplifi the formula. The same formula can be easily modified to calculate the change in draught if the ship passes from dock water of one density to dock water of another.

DWA(mm):FWAtW

25

Load Lines (MAR Rev.

rzl08l02)

Answers Prepared by MCA OOW Nuri KAYACAN

Dec 2012

Fresh water allowance (FWA) The Fresh Water Allowunce (FWA) of a ship is the number of millimetres by which the mean draught changes when a ship
passes from salt water to fresh water, or vice-versa, when the ship is loaded to he summer displacement.

The FWA is found by the formula:

FWA (mm): A Summer


4TPCsw
are considered, the top of the Summer mark and the top of the Fresh mark act as the limits of a scale of density that would appear on a hydrometer (an instrument for rneasuring liquid density).

If the load line marks

The ship behaves exactly S very large hydrometer!


.000 (Fw)

T
FWA

-05

-10 -15

r r rrr-20

r.ozs (sw)

Load Lines (MAR Rev.


r2108102)

Answers Prepared by MCA OOW Nuri KAYACAN

Dec 2012

LOAD LINE MARKS. STARBOARD SIDE


(Dimensions in mm)

lll

300

FWA(mm)

:Asummer 4TPCsw

x= SUMMERDRAUGHT
4g

'Always look forward to summer! '


230

i-230-*l

STATUTORY FREEBOARI)
RD 1.000 (FW)

RD 1.02s (SW)

50
I

LI

W WNA*

Ship. 100 m or less

Load Lines (MAR Rev. 03/01/01)


Answers Prepared by MCA OOW Nuri KAYACAN

Dec 2012

The load lines for the starbourd side of a ship are shown.

Note The spacings between the load lines are measured from the top se-ot one line to the top edge ofthe other! The assigned (Summer) freeboard is measured from the top edge gf the plimsoll line (which corresponds to the top edge of the gq-.r line) to the top edge of he deck line.
NOT TO SCALE
FWA (mm) = PISPL. summer 4TPCsw
X
=

SUMMER DRAUGHT
48

'Afways looJ forward f smner'


I
I

ASSIGNED SUMMER FREEBOARD


RD {.000 (FW}

i<-3oo---'+i
450--+l

50

+{

,
W WNA

I
*

RD 1.02s (sw)

shps 100 m or less

Load Lines (MAR Rev. r2l08l02)

Answers Prepared by MCA OOW Nuri KAYACAN

Dec 2012

LOAD LINES The M. ^S. (Load Line) Regulations 1998 (Amended 2000) require that all UK registered ships be assigned a freeboard and a
corresponding set of load lines to be marked pernanently on the ship's side.

The assigning authority, usually a classification society such as 'Lloyds Register of Shipping' will issue a Load Line CertiJicute.
Important factors that are taken into account include:

* * * * * * * {<

Ship stability and reserve buoyancy; Structural strength; Hatchways; Machinery space ope ings; All openings in the freeboard deck;
lree_iqg norts; Protection of crew; Ship type (A or B).
(and many others also.)

Two ship types are considered:


Tupe A Is a ship designed to cawy only liquid curgoes in bulk (tankers).

Any other type of ship (bulk curuier, contuiner ship, general


c_qlgq qtc.)
Load Lines (MAR Rev. r2l08l02)

Tvpe

Answers Prepared by MCA OOW Nuri KAYACAN

Dec 2012

LOAD LINE CALCULATIONS


SAO

A ship flouts in SW at the summer displacement of 1680 tonnes. If the TPCSW js 5. 18, how much witl the druugh chunge by if the ship is towed o a berth where the density of the water is
1.000
t/m3

Answer

rn moving from SW to FW the ship will experience sinkage


an mount equal o the FWA.

by

FWA (mm)

DISPL. Summer
4TPCSW
87.7 mm

FWA: 1680 :
4x
5.18

The draught will increase by 81.1 mm!

Load Line calculations (MAR Rev. 13/11103)

Answers Prepared by MCA OOW Nuri KAYACAN

Dec 2012

LOAD LINE CALCULATIONS


Introduction

Most ships will be assigned a minimum freeboard and a _criesponding set of load lines. These will be permanently marked on each side of the ship (Certain classes of ship are
exempt from these requirements).

'zones'. Oceans around the world are divided into these zones in terms of both geographical location and time of year (season). By ensuring that the appropriate seasonal load line mark is not submerged at sea in salt water (RD 1.025) the ship will always
have the necessary reserve buoyancy to ensure seaworthiness.

Load lines assigned to a ship correspond to ocean areas

or

perform calculations to determine the maximum amount to load is also important, especially to the ship owner, as the absolute maximum cargo in terms of weight should be carried whenever possible. It is also essential that the ship is never'overloaded', os contravention of the conditions of load line assignment will arise, resulting in the ship being unseaworthy with respect to legislative requirements.

To ensure that the appropriate load line is never submerged at sea, it is essential that the learner has a thorough knowledge of the load line markings, their spacing and dimensions. The ability to

Load Line alculations

(MAR Rev. l3llll03)

Answers Prepared by MCA OOW Nuri KAYACAN

Dec 2012

Answer

DWA (mm) : FWA x (RDpwr- RDor) DWA (mm) DWA


260 x (1016 - 10041 25

Therefore:

= 724.8 mm

The draught will decreuse by 125 mm since the shp s moving into more dense water! Answers need only be to the nearest mm!

Load Line calculations (MAR Rev. 13/11/03)

Answers Prepared by MCA OOW Nuri KAYACAN

Dec 2012

SAO

A ship hus a FWA of 200 mm. Calculute

the change in druught that will occur if the ship proceeds from SW to a berth where the RD of the dock water is 1.018.

Answer

DWA (mm) : FWA x (1025 - RD dock water)


25

Therefore:

DWA (mm) : 200 x (1025 -

10181

25

DWA:

56 mm

The draugh will increase by 56 mm!

Load Line calculations

(MAR Rev. I3ltll03)

Answers Prepared by MCA OOW Nuri KAYACAN

Dec 2012

Answer Qn reuching seqwater of greater density, the ship will be light of


the summer marks as shown below.

MORE CARGO COALD HAVE BEEN LOADED!

-REQ'D (SW)

1.02s

(sw)

To avoid this situation but to also ensure that too much cargo is
never loaded, the amount to safely load can be readily calculated. The uim of the problem is to ensure that on proceeding to sea the ship rises to the desired seasonal loud line murk. This is achieved by considering the Fresh Water Allowance or Dock Water Allowance as appropriute in the culculation.

Load Line calculations (MAR Rev. 13/11/03)

Answers Prepared by MCA OOW Nuri KAYACAN

Dec 2012

Typical load line calculations


possible. If lhe ship is being loaded in water tha is less dense than salt water, such as dock water, then allowance must be made for the ship rising ot of the water on reaching the Sea, salt water density being I.025 tlm3 . Consider the following situation:
the Summer zone in dock water RD 1.012. It can legully load so that the salt wuter wuterline is level with the lop edge of the Summer Load Line.

When loading a ship it is desirable to load as much cargo

as

A ship is louding in

line!

Consider the situution where the officer in churge loads crgo until the dock water wuterline is level with the Summer loud

r.0r2 (Dw)

will be the situation when the ship reaches the sea?


Load Line calculations (MAR Rev. 13lIIl03)

Answers Prepared by MCA OOW Nuri KAYACAN

Dec 2012

3.

culculute the maximum amount thut cun stilt be loaded in dock wuter, ignoring rny allowances for fuel
or other items.
Permitted sinkage (cms)

TPC

Therefore:

Permiffed sinkage (cms) x TpC

lYote thut TPC must be coruected


water!

for the

density of the dock

4.

Muke allowance now for items other than cargo that


must be loaded.

Total that can be loaded Fuel Maximum cargo to load

345.1 tonnes 26.0 tonnes

379.7 tonnes

NOTE Had the gven TPC not been


out us:
w

dock water, the total thut could be loaded would have worked

converted

for the density of

the

16.1 x 27.82

351.3 tonnes;

resulting in the shp beng OVERL0ADED BY 6.2 T0IY]YES!


Load Line calculations (MAR Rev. 13/11103)

Answers Prepared by MCA OOW Nuri KAYACAN

Dec 2012

Example I A ship hus a summer loud draught of 5.80 m, FWA 140 mm and TPC of 21.82. The ship is loading rt u berth in dock water RD 1.007 und the present druught s 5. 74 m. Calculate the maximum mount of cargo that can still be loaded for the ship to be ut the Summer loud line murk on reaching the sea ullowing for 26 tonnes of fuel still to be louded prior to suiling.
The following procedure and layout should be followed exactly.

7.
DWA

Calculute DWA (to the neurest mm).


(mm)

140 x (1025 - 1007) 25

100.8 mm = 101 mm

2.

Always start with the required load line draught und work as follows:
Required Summer draught Required draught Initial draught Permitted sinkage

Culculute the 'permitted sinkuge'

in dock wleF.

DWA

(I.025)
(1.007) (1.007) (1.007)

5.800 m +0.101 m 5.901 m 5.740 m 0.161 m

Load Line alculations (MAR Rev. 13ltll03)

Answers Prepared by MCA OOW Nuri KAYACAN

Dec 2012

Answer

1.

Calculate DWA

DWA (mm)

100

(1025 - 1002t 25

:92

mm

2.

calculute the 'permitted sinkage' in dock Required Winter

wuteF.

DWA

draught (1.025)

Required draught Iniiul druught Permitted sinkage

4.320 10.092 (1.002) 4.412 (1.002) 4.300 (1.002) 0.112

m m m m m

3.

Calculate the maximum umount that can still be Ioaded in dock wteF.
Permitted sinkage (cms)

TPL
(cms) x TPC

Therefore:

: Permitted sinkage

Total thut cn be loaded 236.5 tonnes

Load Line calculations (MAR Rev. 13llrl03)

10

Answers Prepared by MCA OOW Nuri KAYACAN

Dec 2012

SAO A ship is flouting in dock water RD 1.002 at a draught of 4.30 m. How much more cargo must be louded to ensure that the ship will be at the Winter load line murk gven that the Wnter
druught corresponding to the winter displucement is 4.32 m and the TPC is 27.60 and the FWA is 100 mm.

Note that the TPC value given will ulways be the one that corresponds to salt wuter for the waterline which is beng loaded
to.

Load Line calculations

(MAR Rev. l3llrl03)

Answers Prepared by MCA OOW Nuri KAYACAN

Dec 2012

X:

0.143 m

0.12 m

Port WL

(Not to scale)

3.

sketch. Summer draught Line thickness

Starting with a known druught (Summer) calculute the draught on ech side by applying the distunces in the

STBD
6.860 -0.143 +0.040 6.7 57 m

Draught each side

4.

Culculate initial mean draught.

Initial mean draught (RD 1.006) _ 6.715 + 6.757

6.736 m

Load Line calulations (MAR Rev. 13/1 Il03)

t2

Answers Prepared by MCA OOW Nuri KAYACAN

Dec 2012

Sometimes a question may be a little more difficult whereby a knowledge of the load line dimensions is essential. It is essential that a sketch he drawn to fully understund what is being asked!

Example 2 A ship is floating in dock water RD 7.006. The wuterline to port is 12 cm below the lower edge of the '^S' mark and on the starbourd side is 4 cm above the upper edge of the 'W' mark. If the summer displacement is 21620 tonnes (correspondng to a draught in salt water of 6.86 m, TPC 18.6), how much cargo remains to be loaded to ensure that the ship will be ut the Winter mark in salt wteF.

7.

Identify the load lines that re mentioned in the question ('S' and 'W' in this case); sketch them and
enter all known dimensions, calculatng necessary.

them

aS

Thickness of the

lines:

25 mm (2.5 cms; 0.025 m)

Distance between Winter and Summer load lines (X):

Summer draught

48

6.86
48

0.143 m

2.

Druw sketch (Port or Sturboard side: t does not


matter!)

Load Line calculations (MAR Rev. 13/1 Il03)

11

Answers Prepared by MCA OOW Nuri KAYACAN

Dec 2012

5.

FWA (mm) - DISPL.

Culculate DWA (in this case FWA must Jirst be calculuted.

Summer 4TPCsw
25

_ 6T620 _
4 x 18.6

290.6 mm

DWA (mm):290.6 x (1025 - 1006):220.8 mm x221 mm

6.

Calculate the 'permitted sinkuge' in dock water


Required Winter draught Required draught Initial draueht Permiffed sinkage
(1

DWA

.025)
(1.006) (1.006) (1.006)

6.717 +0.221 6.938 6.736 0.202

m m m m m

7.

Culculute the maximum mount that cun still be loaded in dock water.
Permiffed sinkage (cms)

TPC

Therefore: w:
NOTE

Permiffed sinkage (cms) x TPC

Steps 5-7 apply to all load line questions and should be strictly

followed!
Load Line calculations (MAR Rev. 13/11/03)
13

Answers Prepared by MCA OOW Nuri KAYACAN

Dec 2012

7.

The hydrostatic particulars for a ship give the following data for the Summer
mark:

Displacement 35800 t Draught 12.0 m Freeboad 2400 mm TPC 42 The ship is loading up river in dock water RD 1.012 and at a stage in loading the freeboards to port and starboard are 2600 mm and 2580 mm respectively. Calculate how-much cargo to load so as to be at the tropical marks in salt water if 150 tonnes of fresh water have to be taken on board before sailing and 50 tonnes of fuel are to be consumed on passage down river to the sea. (2184 t)

8.

Sketch a hydrometer suitable for use on board a ship and describe how and under what circumstances it should be used.

9.

ship is loading in a dock up-river in water RD 1.004 and is required to proceed to an explosives anchorage at the mouth of e river in order to complete loading a further 450 t. The summer freeboard in SW is 4990 mm and the winter penalty is 190 mm. FWA is 180 mm and TPC for the loaded sunmer draught is 29. Calculate the freeboard which the ship must be at on leaving the dock bertt, if it is to be at the winter load line on reaching the open sea if it is anticipated that 60 tonnes of fuel will be consumed on passage from
the berth to the

sea'

er66

mm)

10.

A small ship has a swnmer displacement of 6220 t, summer freeboard in SW of 1072 mm which coresponds to a srrmer draught of 5.808 m. When

floating at the sunmer load line the waterline length and breadth are 95.0 m and 13.0 m respectively, Cw being 0.750.

(a) (b)

If the ship has to load to the tropical maks in salt water find the draught to which it should be loaded in a port where the dock water
RD is 1.008.
(6.0a m) Sketch the load line marks for this ship as they would appear on the

Port side indicating all variable and fixed dimensions showing all
calculations as appropriate.

MAR.LOADLINE CALCULATIONS.tutorial MAR Rev l2ll2l00


2

Answers Prepared by MCA OOW Nuri KAYACAN

Dec 2012

SHP

STABLTY I01

Answers Prepared by MCA OOW Nuri KAYACAN

Dec 2012

If B is plotted for several small angles of heel it may be assumed that it follows the arc of a circle centred at M.

where v is the volume of th9 transferred wedge of buoyancy and V is the ship's volume of displacement.

is termed the metacentric radius an the height of the initial transverse metacentre (KM) may be glgglqled thu;

BM

KM=KB+BM

Initial Transverse Metacentre (MAR Rev. o7/0r/01)

Initial Transverse Metacentre (MAR Rev.

07lovor)

('
Answers Prepared by MCA OOW Nuri KAYACAN

Dec 2012

Calculatins KM for box-shaped vessels


For a box-shaped vessel on an even keel:
and;

SAO Prove that the

KM of a box-shaped vessel changes wth draught as shown below for the range of draaghts 1.00 m to length: 100 m 15.00 m given that: breadth: 20 m.
ase the formulae to jnd values of KB and BM and lhen sum these toJind the correspondng KM values.

where I is the moment of inertia (second moment of area) of the WPA and V is the volume of displacement of the box-shaped vessel.

zu

For a box-shaped

vessel: I: !3
12

where

and

are the length and breadth of the WPA

respectively. Therefore:

Thus:

Wro*=draught *

lzLB,d

IJB3

Note that this formula can obviously be simplified further!

Initial Transverse Metacentre (MAR Rev. 0710t/01,

Initial Transverse Metacentre (MAR Rev.

07/ou0t)

Answers Prepared by MCA OOW Nuri KAYACAN

Dec 2012

Answer The values for KM are shown below having been calculated

The graph is plotted at the end of these notes.

asing:
KB:

KM:KB +BM

where:

SAO
=

draught

and
BM (m)
33.33 {6.67
11.11

BM

B-

)raught (m
1

KB (m)
0.5
1

KM (m)
33.83 17.67
12.61
f

1.5

ase the graph you have dran to deermne thefollowing: (a) The minimun value of KM and the draugh at whch it occurs; (b) The range of draughts at whch the vessel wilI be unstable; (c) The righting moment when the box is heeled to an angle of 5" when the uprght draughl S 3.00 m in salt water (RD 1.025). KM
REDUCING
I

4
5 6 7
8
9

2 2.5
3

3.5

4
4.5 5 5.5 6 6.5 7 7.5

{0
11

12 13 14 15

8.33 6.67 5.56 4.76 4.17 3.70 3.33 3.03 2.78 2.56 2.38 2.22

0.33 9.17
8_56

8.26

8.17
8.20 8.33 8.53 8.78 9.06 9.38 9.72

KM

INCREASING

SAQ Plot the valcs of KM with lhe X'axs labelled 'draught' and the Y-axs labelled 'KM' and 'KG'. PIot a value of KG = 9.00 m
on the graph.
Initial Transverse Metacentre (MAR Rev.
Initial Transverse Metacentre (MAR Rev.

o7l0l/ot)

07/0t/0t)

Answers Prepared by MCA OOW Nuri KAYACAN

Dec 2012

Answer
(a)
(b) (c)

F'ACTORS AFX'ECTING
The minimum value of KM = 8.00 m and occars at draught= 8.00 m. The range of draughts at which the vessel will be unstable s between 5.20 m and 12.50 n At a draught of 3.00 m KM was calculated to be 12.61

KM

KM 12.61m

KG

GM 3.61m

9.00 m

Displacement:L x B x d x densitY
Displacement: 100 x 20 x 3,00 x 1.025
Displacement = 6150 tonnes
BI

GZ=GM

x Sin0
Sn 5" from the

GZ:3.6lx

GZ:0.31463....
Therefore:

Righting momen= GZ x Displacemenl Righting Moment=0.31463.. x 6150 Righing moment= 7M-

buoYancY is transferred B to move futher -9gPng

Initial Transverse
Metacentre

(MAR Rev.

Initial Transverse Metacentre (MAR Rev.


oT

07/0v01)

lov0r)

Answers Prepared by MCA OOW Nuri KAYACAN

Dec 2012

2.

Drausht (I)isnlacement)
M

Consider the formula:

BB.:vxbb. I V
-t

a!.rhe--e4--qa1gl1t !he-y9_lg19 o{ !b" transferred wedge o{ buoyancy (v) represents a smaller part of the total volume of -disFlacernnTcif 6 ahip (v) than-at-the iight draught.

Initial Transverse Metacentre (MAR Rev. 07/0t/01)

Answers Prepared by MCA OOW Nuri KAYACAN

Dec 2012

:--

TRANSVERSE STATICAL STABILITY

momentarilY

at res1l.

It is the relative positions of the centre of gravity (G) and the centre of buoyancy (B) as the ship is heeled to a particular angle that determines how stable a ship is.

SAO
WitI his ship heelfurther or return to the upright if the erternal force s remov ed?

Transverse statical stabilitY (MAR Rev. 03/03101)

Transverse statical stability (MAR Rev. 03/03/01)

Answers Prepared by MCA OOW Nuri KAYACAN

Dec 2012

Righting lever (GZ) increases to some maximum value and Answer


The ship will return to the apright.
then decreises ui the ship pogressivIy heels further.

The horizontal separation of the lines of action of Wf and


g

i is suh

that a r g hl ng !e-v

r-

(G-1) q9w e1ists,

Transverse statical stabilitY

Transverse statical stabilitY

(MAR Rev. 03/03101)

(MAR Rev. 03103/01)

Answers Prepared by MCA OOW Nuri KAYACAN

Dec 2012

Rishtins moment

W
gv given
a_ngle

----

The righing momenl

9f heel is found

the ooint of intersection of the lines of action Is defined as**-l-_--_*".

q@-ql}9'-^!_,p10' heel approximately).

- "

ilte_iil9l

fo
X

It

RIGHTING MOMENT = GZ

DISPLACEMENT

The rightng moment at any angle of heel represents the


in 'still water' conditions.

-1gglg9-'rp

It's height is quoted in relation to the keel - KM.

Transverse statical stabilitY

(MAR Rev. 03103101)

Transverse statical stability (MAR Rev. 03/03/01)

Answers Prepared by MCA OOW Nuri KAYACAN

Dec 2012

ry

Answer

I lb_ v_qt-q3________________!-a1!1_1-99-b-e!'w99_L!-9-"hip's (G) and the metacentre (Io. ':i 'l l / (' )

9e1ty '')

of gravity

The ship would be less stable. Consder the same ship wth G now at G,.

(-,)

Rghng tever (GZ) has reduced to GrZ, as a result of the upward movement of the centre of gravity (G).

SAO

Therefore, available righing moment wil.l also be


reduced.

If

the centre of gravily (G) of the ship shown wos ighu, would the shp be more ol les stable?

Similarly, if G was lowered the righting lever (GZ) (and available rightng moment) would be increased.
Transverse statical stability

Transverse statical stabili

(MAR Rev.

03103101)

(MAR Rev. 03103/01)

Answers Prepared by MCA OOW Nuri KAYACAN

Dec 2012

laD&_giliurum
is i stable equlbrum if, when heeled by an external force to a small angle, it returns to the upright when the force is removed.

A ship

In triangle

GZM:

Sin

0:

: HTP
OPP

GZ GM
a

Therefore:

GZ=GM

Sn0

Having found GZ:

RM:

GZ X DISPLACEMENT

Note

The formula for GZ can only be used atsmill angles of heel.


Transverse statical stabilitY (MAR Rev. 03/03/01) Transverse statical stability (MAR Rev. 03103/01)

(,r'"

/g)

In this condition the ship has apositve

iniliql GM.
t0

Answers Prepared by MCA OOW Nuri KAYACAN

Dec 2012

Answer

Possibly.

heels furher over the centre of (B) move wiII outward as lhe underwaler volume buoyancy clanges shape. Provded that tle centre of baoyancy can move sulftciently outboard to attain new postion

As the shp

vertically below G the capsizing lever will disoppear and the shp wll come to rest at an angle of loII. If the centre
of gravily wus very hgh then the ship woald capsize.

SAO

[hen the vessel comes to re! at on qnglg of loll will- !!1e ot a small or a large angle?
Transverse staticat stabilitY (MAR Rev. 03/03/01) Transverse statical stability (MAR Rev. 03/03/01)

be.

l2

Answers Prepared by MCA OOW Nuri KAYACAN

Dec 2012

Answer

Answer
centre of grauiy (G) and a postive righting lever (GZ) take e ect to right the shp back to the angle of loll.

M. It has moued to a poston on the same yerhcal as G -ai consdered to be at the same posion as G. II is
may be
ryow lermed

I['|he me-tagentre (M) is considered, it is now no longer a

The centre of buoyancy (B) will move outboard of the will

angk.

@!. fhere

'pro-melacentre2 or a |novng metacenlre' re the ship must now be lying at a 'large'

SAO

If he ship s heeled beyond the angle of loll what will


happen?

Transverse statical stability (MAR Rev. 03/03/01)

l3

Transverse statical stability (MAR Rev. 03/03/01,

Answers Prepared by MCA OOW Nuri KAYACAN

Dec 2012

Answer
Eventually the centre of huoyancy (B) will move oatboud of the centre of gravity (G) and rghtng levers wll become positve to right the shp back o some ndetermnate small angle.

Transverse statical stabili (MAR Rev. 03/03/01)

Transverse statical stabilitY

t6

(MAR Rev.

03103/01)

Answers Prepared by MCA OOW Nuri KAYACAN

Dec 2012

THE GZ CURVE

- AD is

brigin at A.
radian).

a line drawn as a tangent to the

GZ cqve which

emanates flom thg

AD cuts a

[ine drawn yertigatly upwards from the heel scale at 57.3' (one

* BC

is a line very close to the origin at angle 0'.

E
(57.3.)

Triangles

ABC

and,ADE.:j1,li.

( ",..n

... DE:BC / AE AC -'


GZ=GM
and Sin 0 is approximately equal to in radians and 57.3': I radian.)

x Sin 0
is measured

0. (where

0.

Consider, for example SinZ" :0'0348994; 2o expressed in radians = 0.034904.

DE =

lo"

GMg^

.'.

DE: GM
t7

Transverse statical stabilitY (MAR Rev. 03/03/Ot)

Answers Prepared by MCA OOW Nuri KAYACAN

Dec 2012

CT]RVES OF STATICAL STABILITY FOR VARYING CONDITIONS OF STABILITY

Fisure 2

1.

lhe
GZ

s!ip- ! now heeled by an external force to a small angle


is positive, which

of

,/

inclination.

condition

will

act to right the ship when the force is

A ship is in a stable condition of stability if, when heeled by an ixteini 1orci n S#!u*t-9r to a small angle of-nclination, i
returns

removed.

tahd

when theforce is removed'

iorrj".
upright.

the ship shown being progressively inclined from the

Fieure I

KM - KG: GM; which isposiive'


GZ is zero.

A typical curve of statical stability for a stable ship will be shown. Figures I and 2 are related to the curvo as indicated'

as

GZ curves for different conditions of stabilitY

GZ curves for different (MAR Rev. 18/09/01)


conditions of stability

(MARRev. t8/09/01)

Answers Prepared by MCA OOW Nuri KAYACAN

Dec 2012

Crve of staticat sabtv for a STABLE sHlP


o.7 0.6 0-5 o.4 0.3

2. '
A ship

Curve of statical stability for a shin in a neutral


condition of stabilitv

s in a neutral condition of Stahilia f' when heeled hy an external force in still iater to a small angle of inclinaton,

it comes to rest a Some indeterminate angle of heel witln ryg!!4! 8! ! of


upright.
i n c l n

ati o n.

GZ
(m)

o-2
0.1 0 -0.'
-o.2

Consider the ship shown being progressively inclined from the

Figure GZ

[Y - 5c:
is zero.

o,

GM:0

-0.3

G M

wf

GZ curves for different


conditions of stability (MARRev. t8/09/01)

GZ curves for different conditions of stability

(MARRev. l8/09/01)

Answers Prepared by MCA OOW Nuri KAYACAN

Dec 2012

Fisure 2

Figure

Bf

angles of heel. Jh9 ship_ls now heeled beyond small GZ becomes positive and the curve now departs from the iangential line drawn from the origin. The initial transverse metacentre no longer applies to the ship since it is now heeled -to a large angle.

I
K

G M

( ii
(

wf wf

A pical

curve of statical stability for a ship in a neutral condition of stability will be as shown. Figures 1,2 and 3 are related to the curve as indicated.

GZ

curves for different

GZ curves for different

(MARRev. r&l09l0l)

conditions of stabilitY

(MARRev.

conditions of stability
18/09101)

Answers Prepared by MCA OOW Nuri KAYACAN

Dec 2012

Curve of stAtleal stabilitv for a NEUTR


0.3 o.25

3.

'

Curve of statical stability for a ship in an unstable condition

A ship is n an
e_rylernal
I

o.z
0.'t5

unstable condition if, when heeled by an in still water to a small ansle, it continues to force when tle externalforce is removed. lge!yrther
Consider the ship shown being progressively inclined from the
upright.
l

(m)

GZ

o'1

Fig. I

o.o
0 -0.05

Fisure do

r - -r- - L - J - - - \ r r r r

I ,b

-o.l
-0.'15

r rrrrY _ _l- trrtl


lIll

\.b

KM - KG: GM; which is anegative


GZ is zero.

value.

\r ts - f - - _ - -

Heel(deg.) G

Note
which the GZ curve initially follows.
Since GM

M
0, the x-axis of the graph is also the tangent along
B

(>(

wf

GZ curves for different conditions of stability (MAR Rev. 18/09/01)

GZ

(MAR Rev.

curyes for different conditions of stability 18/09/01)

Answers Prepared by MCA OOW Nuri KAYACAN

Dec 2012

Fisure 2

Fisure 3

The ship continues to heel over until B attains a position vertically below G as shown. It is now that the ship comes to restat anangleofloll.
When lying at an angle of loll:

-GZ

i negative; itrepresents a capsizing |ever'

,9h?:lfuther over!)

(If the external force is now removed the ship would continue

GZ is zero.
The angle of loll is a large angle of heel since the line of action of the buoyancy force (Bf) is.no longer passing through the initial transverse metacentre (M).

I
I

Z M

G I

wf

GZ curves for different conditions of stabilitY

GZ curves for different


conditions of stability (MAR Rev. 18/09/01)

l0

(MAR Rev. 18/09101)

Answers Prepared by MCA OOW Nuri KAYACAN

Dec 2012

Fieure 4
-outbo4rd

If thg s!ip_ is heeled further by an external force B


of G.

-';l- ]r

{'ir;l

i :i :'

Curve of statical stabilitv for an UNSTABLE ship

moves
the

0.25

trt -+--t--F0.15
0.1

GZ is now positive which will act to right the ship back to


angle of lolL Bf

Fig.3
(0,-ou)
I

GZ
(m)

0-05 0

i-,
[ig.2

-ri
,

n,;

G'. Z

-0.05

Co: |i iN. !__r H7


I

il.';l
+

t 1
{o

\
l l9v +),,i
l
I

li'l

itial GM GM
tllttl;

lllli

\P

a"l

. . i

-o.1

--r-f-T-T--t----r'
ttltt

tt- l

l-

' ttti
.

__r_l
,......'

I__i
'

-l
'' .'
.. .'j

-0.15

ll,
K wf
Note

Heel (deg.)

A typical

curve of statical stability for a ship in an unstable condition of stabili will be as shown. Figures 1,2,3 and 4 are

related to the curve as indicated.

Since GM is negative, the tangent along which the GZ curve initially follows runs below the base. As the vessel heels over to larger angles of inclination the GZ curve departs from the tangent and where it crosses the base is the angle of loll (approximately 11o in this case). In theory this could be to the port or the starboard side since G is assumed to be on the
centre-line whereby port and starboard moments are equal.

GZ curves for different conditions of stability (MAR Rev. 18/09/01)

GZ curves for different conditions of stability (MAR Rev. 18/09/01)

t2

Answers Prepared by MCA OOW Nuri KAYACAN

Dec 2012

4. ,
GGr'

Curve of statical sta

Fisure 2
the centre-line becomes less and less.

When a ship is listed th9 celtre of gravity of the ship is off the centre-line to port or starboard by a distance we have termed

A'

s!p lit over the capsizing lever caused by G being negative.

off

GrZ is
1

Fiqure

Consider the ship shown that has G off the centreJine to starboard that is initially in the upright condition-

GG"

repres ents a capsizng lever; a negatve value of GZ-

GZ curves for different

l3

GZ curves for different


conditions of stability (MAR Rev. 18109/01)

t4

(MAR Rev.

conditions of stabilitY
18/09/01)

Answers Prepared by MCA OOW Nuri KAYACAN

Dec 2012

Figure 3

X'isure 4

GZ is zero.

ll

\ru
I

A typical
indicated.

curve of statical stability for a listed ship will be as shown. Figures l, 2, 3 and 4 are related to the curve as

GZcurves for different

l5

(MAR Rev.

conditions of stabilitY
18/09/01)

GZ curves for different conditions of stability (MAR Rev. I8/09/01)

t6

Answers Prepared by MCA OOW Nuri KAYACAN

Dec 2012

Curve of statical stabilitv for a LISTED ship


o.25 o.2 0.15

0.t

GZ
(m)

0-05 0 -0.05

7,.;-

{.1
-0.15

Hee! (deg.)

Note

At

GZ value is negative by an amount equal to the distance that G is off the centre-line (GGn)- This causes the base of the graph to be dropped vertically to coincide with the new origin - the green line being shown as the base of the graph. The initial GM used to produce the tangent to the curve
0o heel, the
at the

origin is measured from the new base as shown'

The angle of list is identified as the point on the curve where it crosses the original base ofthe curve as shown.
GZ curves for different conditions of stability (MAR Rev. l8/09/01)

GZ curves for different (MAR Rev. l8l09l0l)


conditions of stabilitY

17

l8

Answers Prepared by MCA OOW Nuri KAYACAN

Dec 2012

Asaihip-tr-eeliR-cn5tamlymvEs;-if 's-p-oIitinis-depenctent
on:

PRODUCING A CTIRVE OF'STATICAL STABILITY


the displacement (draught) of the ship; the angle ofheel at anY instant.

If the figure is considered:

* *

Sine

0: OPP
rryP

Sine

0:

Correction to

KG

KN
x Sine 0

Therefore:
and:

Corretion to

KN: KG

GZ

KN - (KG Sine

0)

to allow GZ values to be
It is usual that l5o intervals.

determined for any value of displacement and KG. Sometimes the values may be tabulated.

Cross curves of stability (KN curves) are provided by the builder

KN

values are given for angles of heel at 10" or

An example of such values is given;

KN Curves (MAR Rev.


03/0r/0t)

KN Curves (MAR Rev.


03/01/0r)

Answers Prepared by MCA OOW Nuri KAYACAN

Dec 2012

DISPLACEMENT
15000
14500 14000

(t)

Procedure for constructing a cuve of statical stabilitv


1.72 2.98 4.48 1.73 2.98 4.51 r.74 2.98 4.55 1.15 2.99 4.58 L.77 3.00 4.62 1.78 3.03 4.63 1.78 3.05 4.65 1.80 3.12 4.70 1.82 3.15 4.75 1.83 3.19 4.79 1.86 3.23 4.83
1.93 3.28 4.91

3500
13000 12500
12000

11500
11000

10500 10000 9500 9000 8500 8000 7500 7000 6500 6000

s500
5000

2.00 3.36 2.O5 3.43 2.to 3.52 2.17 3.62 2.22 3.70 2.32 3.85 2.42 4.00 2.57 4.15 2.72 4.32
ii

s.00
5.04

s.10
5.18 5.25

5.J)
5.45

5.5s 5.6s

5.72 6.48 5.79 6.58 5.85 5.68 s.90 6.73 s.93 6.78 s.98 5.83 6.O4 6.88 5.10 6.93 6.15 6.98 6.18 7.02 6.22 7.07 6.25 7.tt 6.28 7.t8 6.32 7.20 6.36 7.22 6.38 7.24 6.40 7.26 6-43 7.21 6.48 7.28 6.53 7.29 6.58 7.30

6.91 7.05

6.9s 7.08 7.00 7.10 7.08 7.13


7.14 7.16

1.

Determine the ship's displacement and effective KG for the condition being considered (effective KG being that taking
into account free surfaces in tanks).

7.18 7.20 7.25 7.30 7.35 7.40 7.50 7.55 7.60 7.65 7.70 7.70 7.70 7.68 7.66

7.18 7.20 7.22


7-24 7.27 7.30

2. 3.
4.

From hydrostatic data find value of


displacement.

KM for ship's

Find GMrr,o using:

GM=

KM-KG
KN value
in metres

7.45 7.35

7.40

7.4t
7.42 7.46 7.50 7.51 7.52 7.51 7.50

Enter KN tables (or curves) and obtain for each angle of heel given. Using: values for angles of heel given. Plot the GZ values.

5.
6.
7.

determine

GZ

JLLAND FORECAS |TLE O NLY.

tcvALUEsARbmrt IATED
IND TDrcD TRIM
ti'l

FORVESSI )LON, \N EV EN KE EL

Note

values are tabulated as shown interpolation for displacements other than those stated may be done, but it should be Lorne in mind that the rate of change of KN will not be linear.

when

KN

Before joining all the points on the curve construct a vertical at 57.3" and from the base upwards mark off the value of the effective GM (using the GZ scale). From this point draw a straight line to the origin of the curve to be drawn. This will indicate the initial trend of the curve at small angles of heel and will assist in sketching the actual curve between the origin and the first plotted GZ value.

(GZ and GM are closely related at small angles of heel)


KN Curves (MAR Rev.
o3l01/01)

KN Curves (MAR Rev.


03/01/01)

Answers Prepared by MCA OOW Nuri KAYACAN

Dec 2012

|ntct stabilitv reqirements - M.S. (Load Lne| Regulalons


1998

Procedure to verifv that a ship's loaded condition comnlies with legislation req uirements
Consider the following example: ship has a displacement of 12000 t,KG 8.22 m and a KM of 8.54 m. Using the KN values provided determine whether the ship's loaded condition complies with the requirements of the M.S. (Load Line) Regulaions 1998.

GZ
(m)

l.

Determine the GZ values. KG HEEL


i 8.22 0

KM i8.s4
t2
1.78

GM
30

0.32 40 6.04 5.28 0.76

20 3.05 2.81 o.24

* 't * * rr *

Area 0o to 30o to be not less than 0.055 m-r; Area 0o to Xo to be not less than 0.09 m-r; Area 30o to Xo to be not less than 0.03 m-r; Xo is equal to 40o or any lesser angle at which progressive down-flooding would take place; Maximum GZto be not less than 0.20 m and to occur
at an arg|e ofheel not less than 30o; Initial GM to be not less than 0.15 m.

KN (m) 0.00 (KG Sin Heel 0.00 GZ(m\ 0.00

4.6s
4.11

t.7r
0.07

0.54

50 6.88 6.30 0.58

60 7.20 7.12 0.08

75

7.20 7.94
-0.74

2.

Plot the GZ curve. Obtain a value for 10o heel so thatGZ values are available at
10" intervals up to 40o heel.

3.

Using Simpson's rules calculate the areas under the curve (0o- 30o and 0"- 40o and 30'- 40')

KN Curves (MAR Rev.


03/0r/ot)

KN Curves (MAR Rev.


03/0t/01,

Answers Prepared by MCA OOW Nuri KAYACAN

Dec 2012

Area 0"- 30"

Intctstahilitv reqlirements - Deoartment of

the Envronment

Heel

i GZhrdl

i SM

i Area Fn.

and Regions

Area:3/8 x
Area 0"- 40"
Heel
0

10157

.3

x I.47

0.096 m-r (0.055)

SM
0 0.07 0.24 0.54

Area Fn.

I
4 2 4

l0
20:
30i 40i

0.28

04
2.L6 0.76

0f
The ratio of the area under e righting nm (GZ) curve to that under the wind heeling arm curve to be nol less lhan 1.4. These areas are to be measured from the upright position to an argle of heel not exceeding either the angle ofprogressive down-flooding (0f) or the second intercept ofthe righting arm and heeling ilm crrves whchever is lesser.

0.76i

Area: l/3 x Area:

10/57.3 x 3.68

0.214 m-r (0.090)

Area 30"- 40"


0.214 - 0.096

0.118

m'r (0.03)

Minimum GM to be nol less than 0.30 m.. Minimum GZ for any angle of heel
&,

Max GZ and angle at which it occurs Satisfies the requirements. Effective GM - KG: GM;
8.54 - 8.22

up to the angle ofprogressive down-flooding 0 or; angle of maximum righting lever, or;

l!",whichever Mnimum GZ=0.5 x 0.3 Sin 0.

s leasl to be given by:

KM

0.32 m (0.1 5)
03/01/01)

SHIP COMPLIES

Angle ofsteady wind heel not to exceed l5o. (Based on a wind velocity of 51.5 m/sec (100 Knots)

KN Curves (MAR Rev.

KN Curves (MAR Rev.


03/0r/0r)

4 Answers Prepared by MCA OOW Nuri KAYACAN

Dec 2012

ship in a stable condition heeled to a small angle

Z
B1

Answers Prepared by MCA OOW Nuri KAYACAN

Dec 2012

SHP

STABLTY I01
(_

Answers Prepared by MCA OOW Nuri KAYACAN

Dec 2012

CENTRE OF GRAVITY (G)


we

Effect of shiftins
the vessel and

weisht already on board

parallel to and inthe same


of the
w

The position of G will not move as the ship heels (provided that *ggtl'-ggggl':4-te-e1"--ryiihln}-g-]iP j.{alwayas verlicallv downwards!

.:wxd w
I -

0.

'd' is the
displacement

distance through

which the weight is s-tftedl---

'w' - (A)

i-t*

bp]

K
The verlcal posilon oJ G s expressed n terms of metres above the keel (KG).
weights are -4[!g! :-L. gL ]9 ryLq d- olJi grye J!--G- w i\ move. Whenever G is caused to move the 'shift of G' must be

wten

SAO

-calculated.

ship displaces 5000 and has an iniliol RG of 4.5 m. Cqlcaae |he Jnal KG i a weghl of 20' t, moved verlcally u.pi,ard from the lowqr hold (Kg 2;0.m) o':he rypper dech, ([g
6.5 m)

Centre of gravity/buoyancy

(MAR Rev. 03l0ll0l)

Centre of gravity/buoyancy (MAR Rev, 03/0t/01)

(_'
Answers Prepared by MCA OOW Nuri KAYACAN

Dec 2012

Answer

Effect of loadinq a weisht

'GG--a-'I*! = 20

w
=

s000

(6'5 - ?'0),'= 0,018 m

!+ps^lgyqrL@
I

lga494_U9ght-@.

Inal

FINA

GG,

KG

:4,500 m
= 4,578 m

KG

0a!JL
.

.=wxd |
'w'is

KG has increased which would make the ship less stshle'

W+w
een G
g-

ofthe ship and


.weight.

?s

of the

loa^dg

'W' (A) is the ship's

initial

displacement.

SAO

Iower'hold at Kg 3.0 m.

ship displaces 12500 t and has an initial KG of 6.5 m. Calculae the !nal KG ,f 1000 t of cargo is loaded into lhe

Centre of gravitY/buoYancY (MAR Rev. 03/01/01)

Centre of gravity/buoyancy (MAR Rev. 03/0/01)

Answers Prepared by MCA OOW Nuri KAYACAN

Dec 2012

Answer

GG,=wx W+

d=!W!)=0.259m w 12500+1000

from the centre of sraviW of the di

InitalKG GG, FINAL KG

:6.500 m :0:259 m :6.241m

GG.=wxd
I

W-w
aisfahT-bffi-een

KG has'decreased which would make the shp more stable.

-of

id' il-the -ffi-

'w'

is the weight discharged.

shib-ana s ;f.---i[e
_

displacemert.

tW' (A) is the ship's initial - '

SAO

the lower hold (Kg 3;0 m).

ship displaces 18000 t and has an iniinl KG of 5.30 n Calculate he JnaI KG f 10000 t of cargo is discharged from

Centre of gravityuoyancy (MAR Rev. 03/01/01)

Centre of gravity/buoyancy (MAR Rev. 03/01/01)

Answers Prepared by MCA OOW Nuri KAYACAN

Dec 2012

Answer

Moments about the keel

GG:L_!: !W0)=2.875m

Example

W-w

18000-10000

A ship displaces
Load:

10000 t and has a

KG

of 4.5 m.

Iniial

FINAL

GG,

KG KG

:5.300 m
=2.875 m = 8.175 m
less lable.

The following cargo is worked:


170 tat

Kg 6.0 m;

730 Discharge:

tatKg3.2 m.
Kg 2.0 m; Kg 6.2 m.
m.

KG has increased whch makes the ship

68 t from

100 t from

It would be very tedious to do a calculation for every single


weight that was either shifted, loaded or discharged from the ship.

shift:
Calculate the final

86 t from

Kg2.2 m to Kg 6.0

KG.

Lr".tqg-g_._qg!s final KG of the ship. MOMENTS

qboyJ [he

kgel' ge 9e!-!o d9j9rmine

!te-

FINAL KG: SUM OF THE MOMENTS


FINAL DISPLACEMENT
slip-ft)
L9-1d
(+-)

WEIGIII
120.00

(t)

KG

(m)

MOMENTS (cm)
45000.00 720.00 2336.00

(t-m) =

WEIGHT

(t) x

DISTANCE (m)

10000.00 730.00

4.s0
6.00 3.20 2.00
6-,.?9

Load (+) Dscharge

P""l"r"g ()"

68.00
-00.00
-86.00

-36.00
-620.00 -189.20

*Load (+)

D_|.gh3_g_e_

(;)

86.00

. 6.00

56'00
47626.80

F'INAL

0682.00

4.4s9

Centre of gravity/buoyancy (MAR Rev. 03/01/01)

Centre of gravitylbuoyancy (MAR Rev. 03/0/0l)

Answers Prepared by MCA OOW Nuri KAYACAN

Dec 2012

SAO A ship displaces 14200 t and has


Thefollowng cargo s worked:
a

KG of 6.22

m.

Load: Discharge: Shif:


Calc ulate,
th
e

768 t at

Kg

7.20 m;

348 tfrom

Kg 4.62 m;

266 tfrom Kg 5.36


188 Jn al

tfromKg

8:00 mto Kg 3.60

KG,

Answer
WEIGIIT
Ship (+)
(t)

KG (m)
6.22
7.20

MOMENTS
88324.00 1209.60
-1607.76 -1425.76

(t-m)

14200.00

The position oJ B _has nothing iveights within the ship.


The centre
o=|

to do with the disposition of

Load

(+)

68.00
(:)
:348:90_

_D_i_s'l"r"rg-"_

Dhcharge (-)

-256.00 -188.00
188.00

4.62 5.36 8.00 3.60 6.229

*Load (+)

P-ig!,__._c"- (:)

-1504.00
676.80

ioSvndhaanS:****'

(proviled weights do not shift within the ship) as the ship heels bui he cen7ie of buoyan ioistaity'moies ai e shp pitches,

eravjtJ (G) !s qsg-mgd !9 19main in the -same place

FINAL

3754.00

85672.88

For a box-shaped vessel on even keel

KB

is halfthe draught.

Centre of gravity/buoyancy

(MAR Rev. 03/01/01)

Centre of gravity/buoyancy (MAR Rev. 03/01/01)

l0

Answers Prepared by MCA OOW Nuri KAYACAN

bg-@
FR-EE SURFACE EFFECT
The righting lever GrZ, is the same as the GZ that would have existed had G
been raised to Gu.

Dec 2012

\
zv
i
i |

GGu represents the virtual rise of G that results from the free surface effect of the slack tank.

I
,LI

Thus:

Gv

GM isthe solid GM;


G.)VI isthe effective

G!

Z
zr

orfluid GM.

R--.g,
IT
TS ALWAYS

When calculating the GM of the ship it is important that the


effects of free surfaces are considered i.e.

ship has a slack tankas shown i.e [g_tank

it=_only

part full.

THE FLUID GM THAT MUST BE ro r.rE ACCoIINT oF THE REDUCTIoN bEfBnMNED

IN GZ

External forces cause the ship to heel to some angle and liquid in the tank moves from-th+Lh slde to the low side (gg').
This. causes G to move off-t[e-cglge-line to G,-

MOVEMENT WITHIN THE SHIP AS IT IS HEELED

VAL_UES

THAT arsps FROM

LIQUID

As a result of the shift of liquid, righing lever is reducedfrom

97-'o

9,"
Free surface effect

Note When the ship is in the upright condition the centre of gravity (G) will move back to it's original position at G. Hence, it is termed a 'virtual rise' of G since it does not actually move up to
Gv.
Free surface effect (MAR Rev-03/01/01)

(MAR

Rev.03/01/01)

Answers Prepared by MCA OOW Nuri KAYACAN

Dec 2012

virtual rise of G

Answer

GG.,=lb3 x

1.

12Y

dt
ds

JI

Taking momenS about tle keel, calculate the new solid KG.
WEIGHT(t) KG
(m)

MOMENTS
79800.00 76800.00

(t-m)

shipft) FINAL

where: 99, i'J!

yirtua-trise of G in metres;

Carso oil

(+)

I is the tank length;

10s00.00 9600.00 20100.00

7.60 8.00 7.791

ttednk breadfh; at ii t" density of the liquid in the tank; ds is the density of the-water in wnichthe ship floats (.n25 t/fi);and,

tiJ

Calculate the effect offree

surfoce.
30 x 12 x20100

('t,'' "i

56600.00 ."-'''
';

GGv=-!b3-

.!i lz\

dt

203

x 0.86 = 0.856 m

1 ._'.

Since:
it follows

A:Vxds

tha
li(

(r r
('ii;'^j

GGu

l_iiil

12L

Calculate the sold GM and then apply the free surfoce correction to obtain theJluid GM.
dt
KM
SOLID KG
SOLID GM FSE (GGv)
T'LTJID

8.800 7.791 1.009 0.856 0.153

GM

(Answer)

displacement is 8,80 m.

Free surface effect (MAR Rev-03/01/01)

Free surface effect (MAR Rev.03/01/01)

Answers Prepared by MCA OOW Nuri KAYACAN

Dec 2012

Consider the previous example.

alculctions

(, often termed the second moment of must first be determined. suiface liquid arei,-o{thehie
The moment of inertia

t. 2.

Calculate the FSM's using: FSM' (t-m)

Ib3

I2

For a rectangular free liquid

surface: I

lb3

jn4)
t2

Taking moments about the keel, also adding the FSM's in the 'momenls' colmn', calculate he/Iuid KG.

If the value of I is multiplied by the liquid density then a value of 'Free Surface Moments' (FSM's) (t-m) is obtained.

FSM's (t-m;

lb3ldt
12

In the previous free surface effect formula:

Apply thefluid KG value to thefinal KM to obtain thefi'nal fluid GM. KM


FLTJID KG
8.800 8.647

GG,r: J| x
Therefore:

dt

12L GG.,: FSM's

FLUID GM

0.1s3

(Answer)

Displacement
= Sum of moments

Since:

DisPlacement it is evident from the above that the greater the value ofthe free surface moments, the greater the loss of GM (GGv) and the greater the value of the effective KG.

Final KG

Note In tank sounding/ullage tablesfree surface momenls (t-m) may be tabulated for an assumed value of liquid density.If the actual -then the FSM value must be ]qa--{qlty !i different corrected.

In calculating tle effective GM it is us the for free surfaces by incorporating where they must alwaYs beADDED.
Free surface effect (MAR Rev.03i01/01)
Free surface effect

(MAR

Rev.03/01/01)

Answers Prepared by MCA OOW Nuri KAYACAN

Dec 2012

Factors affecting free surface effect If the free surface formula is ion-i?i'ered it is obvious that the breqdlh of the tank is. the most impgrtlnt factor.
If a tank is_subdivided_the loss of

2.

With a sinsle centreJine division

GM

can be greatly reduced'

E
(.'.(

Consider the rectangular tank shown below.

1.

With no subdivision

'"

The volume of liquid that has moved is halved. The distance that the total volume of liquid has moved

iJ ai.o halved.

ry!
If the loss of GM due to the free surface of the tank is 0'64 m and the solid GM was found to be 0.70 m, say, the effective GM would be:

quarter of

single centre-Iine division the FSE is reduced to a


th|e

ornal valub Solid GM FSE (m) Effective GM


0.700 m 0.160 m 0.540 m

0.64:0.16 m

Solid GM FSE (m) Effective GM


Free surface effect

0.060 m
(MAR
Free surface effect

(MAR

Rev.03/01/01)

Rev.03i0ll01)

Answers Prepared by MCA OOW Nuri KAYACAN

Dec 2012

3.

With two subdivisions

; ,a)

9L
3

With three suhdivisions i.e. dividing the tank into four impiimeits, FSE would be reduced to one sixteentl and so
It is usual to subdivide tanks into three compartments at most as

{r==-

the benefit of any further subdivision would improve effective GM by a less significant amount each time (with the exception of product carriers/chemical carriers etc. where a ship is designed to carry a wide range of cargoes at any one time).

The volume of liquid that has moved is now a third. The distance that the to'idlvolume of liquid has moved is also only a third.

With the tank equally subdivided with two longitudinal bulkheads he FSE s retruced Io a niith of he orginal value'

Typical oil lanker lank arrangemenl

0.64

:0.071

Solid GM FSE (m) Effective GM

0.700 m 0.071 m 0.629 m

Free surface effect

(MAR

Rev.03/01/01)

Free surface effect (MAR Rev.03/01/01)

l0

Answers Prepared by MCA OOW Nuri KAYACAN

Dec 2012

Points to note

- Fr*

an 'oil-tight' or 'water-tight' bulkhead ensuring that there is no possibili of liquid transfer. This means that any valves in between the-tq!_ry9tb-e capable of being fully shut'

trrk

to be considered subdivided it must be ftted with

The two tanks shown are exactly the same in size and both hold liquid of the same density but are filled to different levels.

Tlefree surfuce momentsfor each are the Same.

'wash plate' is fitted to prevent damage-to internal tank plating that may be caused by wave action withn the tank'

>/''

Free surface effect (MAR Rev.03/01/01)

ll

Free surface effect (MAR Rev.03/01/01)

12

Answers Prepared by MCA OOW Nuri KAYACAN

Dec 2012

Consider a ship floating upright, G and B on the centreline with a


yr-eighf

'w' on one side-.

The weight ''w' is shifted transversely causin G to mo'e off the centre-line as
shown. The ship lists over, coming to rest with the centre of buoyancy, B, vertically ba;w ah .*ti" of gravity, ,o*ffi .,

;ru
d

List(MAR)

List(MAR)

Answers Prepared by MCA OOW Nuri KAYACAN

Dec 2012

The angle at the metacentre in the right angled triangle the list (termed 0).

GG,M

is

Answer

KM

Tan 0rr.r=
Therefore:

ADJ sr

OPP: GG, GM

KG 6-7 m GM 0.6 m

7.j

GGu:w

xd

GGn- 0x 14 :0.07 m
12000

Tfln0,,o-= GGLto I
-tt

GM

0.07 =
0.6

0.11666

List:6.65"

Tan 0161:

GM
,(

If a weight is loaded or discharged then both the vetical and horizontal components of the shift of G must be considered and th final GM must be used to calculate the final list.

, ,l

For the above formula to be true, the list must be restricted to a small angle, i.e. the metacentre is assumed to be in a fixed poiition *iahTn small angles of inclination.
.l

',

1. Calculate

the final

GGu, and hence GM.

2. Calculate

GGr.

SAO

3. Calculate the list.

A shp intially uprght dsplaces 12000 t and has KG 6.7 m and KM 7.3.m- A weiht o 60 t already on board is shfted 14 m horzontally acoss the deck- Calculate the resulting angle of
ISt.

SAO

shp displaces 6400 t and ha KG 4.6 m and KM 6.5 m, A weght, of 80 t s loaded dn deck at Kg 10.2' m, 6.2 m off the ce ntre-Ine to starbo ard. CaIc ulate the final,: Iist,
List (MAR)

List(MAR)

Answers Prepared by MCA OOW Nuri KAYACAN

Dec 2012

Answer

GGr=w

KM=
Inal

W*w

GGv= 80x

(10.2 + 80

4.0

= 0.069

6400

_ ;- . ] '.l .* SAa',; .' A ship:dsplaces 8000 tonnes, RG 7.60 m and Thefollowing cargo is worked:
:

s intally aprght.

Intial

KG

Fina

GG"(uil
GGu=

GM

6.500 m 4.600 m,

GM

I.900 m 0.069 m 1.83I ry

Load: ',

300 ! atKg 0,60 m,6.1 mto port 'i'; 250 t a Kg 6.10 m' 7.6 m to sbd of CL; Disch:: .,, 50 tfrom Kg 1.20 m, 4.6 m to pott of CL; ,,,, 500 tfrom Kg 12.60 m; 4:6 m to stbd of CL.
:

oJCL;

& W*w

GGr=

80'x t6.2 64A0,+80

0.077 m

Calculate:thefinal angle of list on completon of cargo.

Tanax17:

GGu :0M- = 0.04205


GM'^n 1.83t

List= 2.4" Stbd


Answer
)'|

lt is easier to solve list problems using moments. Take moments about the keel to determine the final KG:

l.

Final KG = Sum of moments about keel (t-m) Final DisPlacement (t)


2. 5.

'-f

t:,

Calculate the final GM:

GM: KM. KG

Take moments about the centre-line to calculate the final distance that G is off the centre-line, GGr:

GGu = Sum of moments about centre-line (t-m) Final DisPlacement (t)


4.

Calculate the list:


List (MAR)

Tan
i i-l

0rrr.: GGn
List(MAR)

Answers Prepared by MCA OOW Nuri KAYACAN

Dec 2012

SUSPENDED WEIGIITS

when the weight is plumbed over the side a larger than normal list will also occui and certain precautions will have to be taken to ensure that the maximum list is restricted to an acceptable limit-

Suspended Weights (Rev.


03/09101)

Suspended Weights (Rev.

03l09lor)

Answers Prepared by MCA OOW Nuri KAYACAN

Dec 2012

of the weight moves vertically up to it's point of at- gr This results in a corresponding vertical movement of G to Grn, causing an increase in KG.
the weight is lifted clear of the tank top the centre

GG,,=wxd Y-

where 'd'-i the istange through which the weight is effectively .$ifted upwards from it's initial stowage position to the derrick
head.

Example A ship has a displacement of 10516 t, KG 8.20 m and KM 9.00 n A weight of 86 t n the lower hold, Kg 3.40 m, is lifed by the ship's heauy lft derrick, the head of which iS 22.00 m above the
keeI.
7X

(a) (b)

Calculate the GM when the weight

s suspended.

Calculate theJinal GM when Ie weght is reslowed in the tween deck at Kg 8.50 m.

I I

Suspended Weights (Rev.

Suspended Weights (Rev.


03109101)

03109/0t)

Answers Prepared by MCA OOW Nuri KAYACAN

Dec 2012

The previous example was very simple as there was no horizontal

GG, ::

86 x (22.00 - 3.40 = 0.152 m

shift of

involved as would be the case when loading or

discharging a weight.

10s16
m m m m

8.200 Iniial KG 0.152 GG, KGwhen weight suspended 8.352 9.000

Wlen lgading or discharging weights using ship's lifting gear the following must be considered:
(t)
the increase in KG/decreqse in GM wlen the weight is suspended at the derrick/crane head;

KM

GM when weight susPendedlQlfu!fu


(2)

This is lhe minimum GM during tle liftng operalion'

the maximum angle of list that wiII occur when the derrck or crane is plumbed over the shp's side.

(b)

To calculate the final GM when the weiglt has been shifted treal as a normal single weight problem simply shif he weightfrom it's initigl stowage position (Kg 3.a0 m) to il's final stowage position (Kg 8,50 m) i.e. ignore the derrick.

GG,,:wxd=
W
Initial

86 x (8.50 -

3.40

0.042 m

10s16

GG, FinalKG KM FINAL GM


Suspended Weights (Rev.
03/09101)

KG

Suspended Weights (Rev.

03l09lot)

Answers Prepared by MCA OOW Nuri KAYACAN

Dec 2012

2.

Consider the movement of the ship's centre of gravity (G) when a weight is lifted offthe quay on one side of the ship and stowed in the lower hold on the centre-line. (For the purpose of this explanation the ship is zol shown to list.)

1.

Derrick picks the weight up off the quay. G moves to G1, directly towards the centre of gravity of the loaded weiht - the point of suspension (g). The movement GG, has two components: GGr; whch cuses an increuse in KG/decrease n GM. GGu: which causes the ship to list.

Derrick plumbed over tle weight.

Suspended Weights (Rev.


03109/01)

Suspended Weights (Rev. 03t09t0t)

Answers Prepared by MCA OOW Nuri KAYACAN

Dec 2012

!t

Derrick swings inboard to plumb stowage position on


the centre-line. G, moves to G, as the weight is swung inboard from g, to gr. Ship becomes upright.

o*n**rrrr*,
towards / E

Tan

0r*.rsr

= GG,n

GM,'*n'

Suspended Weights (Rev.


03/09/01)

Suspended Weighs (Rev.

03l09lol)

Answers Prepared by MCA OOW Nuri KAYACAN

Dec 2012

4.

Weight landed in the lower hold. The weight is refuovgd from the derrick head at g, as it
movGs to Gj-

The following example illustrates a typical list question involving a heavy lift being loaded. The calculation may done by one of two
methods:
(1)

is landed in the lower hold and finally acts at E-Gz

Note that the weight is loaded below the initial centre of

Formula method. Taking momenis-abou tle keel and the centreline,

gravity of the ship, -hence the final position of G, G, must be lower. KG is reduced overall.

Both methods are shown, however, the moments method is much simpler and will be adopted for all other examples in this section.

Suspended Weights (Rev.

Suspended Weights (Rev.

12

03/09/or)

03/09/ot)

Answers Prepared by MCA OOW Nuri KAYACAN

Dec 2012

of he ship's centreline. I the head of the derrick is 27'0 m ibor" lhe keel when topped to it's maximam extent for the I fng op eralo n, calc uI ate :

7.15 m and KM 7.98 n A weight of e quay which is 15.0 m to starboard

Soluton (Method 1|

(a)
GG,,:

To calculate the GM when tle weighl is suspended

Load tle weglt at tle derrick lead.

(a) (b) (c) (d)

yJI
W*w

GG"= 40 x (27.00 - 7.1fl = 0.089 m


8850 + 40 7.150 m

the GM when the weight is suspended;


the maximum angle of list;
the

Iniial KG

Kg

Jinal angle of

list if he weight is placed on deck at 10.40 m, 5.0 m off the centrelne to starboard

MaxmumKG KM MINIMUM GM

GG,

0.089 m
7.239 m 7.980 m

0.741 m (GM when the weght is suspended)

the weght of ballast to transfer between two double bofiom tanks, each having is centre of gravity 4'0 m offthe cenlrelne, to bring the ship uprght'
(Assume

(b)

Calculate the distance that

when the weight is suspended over the quay

G is off the

(CCr)-

centreline

KM rernains constant)
Tan

GGu:wxd W*w
0**us7: GGn
GM,,,,

GGn: 40x15 : 0.067m


8850 + 40

Tan 9rn*.

Lrsr: L067
0.741

0.09042

Maximum list= 5.2" Stbd.

Suspended Weights (Rev.

Suspended Weights (Rev.

t4

03/09/ol)

03/o9iol)

Answers Prepared by MCA OOW Nuri KAYACAN

Dec 2012

(c)

Calculate the final angle of list when the weight is placed on deck. (Simply load tle weight on deck, ignoring the derrick as in a normal single weight problen)

(d)

Calculate weight of ballast to transfer from Stbd. DB tank to Port DB tank.

In tlefinal condition G is off the centreline by 0.022 m (GG*)

GG.,: /- w x d
W+w

GGv:40
7.150 m

x (10.40 - 7'15) 8850 + 40

0'015 m

GG..:wx D

0.022-- w x

I w:0.022x I 8890

8890

24.5 t

Initial GGn

KG

Transfer 24.5 t

Final KG

0.015 m
7.165 m 7.980 m

KM

Final GM

0.815 m

GG, =wxd
Tan 011s7:

W*w

GG-=40x5

=0.022m

8850 + 40

GG.n Tan O"rrr: 0.022 : 0.815 GMor*nr.


FnaI

0'02700

list:

1.5" Stbd.

Suspended Weights (Rev.

l5

Suspended Weights (Rev.

I6

03/09/ol)

03/09iol)

Answers Prepared by MCA OOW Nuri KAYACAN

Dec 2012

Solution Method 2l
(a)

(c)

Calculate the Jinal angle of list when the weighl is

To calculate the GM when tle weiglt is suspended'

Load the weglt at tle derrick lead. Take momenls aboul the keel.
weisht (t)
8850-0

weisht (t 8850.0 40.0

Ks

(m) moments (t-m' 63277.5

7.15

Ks (m) noments (t-m


7.15
632',77.s

,8890i

0.40
7.165

415.0 636935-

KM KG GM

7 980

7 165 0.8I5

KM
KG GM

7.980 7.239 0.741

40.0

27.00

8890;0-- +.239-

1080.0

Take moments about the centreline to determine


Dist off Port stbd weisht (t) CL(ml moments (t-m moments (t-ml 8850.0 40.0 0.00
0.0

GG*

-643515

GM when

weght suspended 0.741 m (Mnimum

GM)

0.0 200.0

Take moments abot the centreline to determine


Dist off weisht (t) 8850.0 40.0

GGff
=

500
'

8890.0

0.02>

200;0'

Port
(t-m

stbd
noments (t-m

CL(m) moments
000
0.0 I5.00

00 _(0J600.0

Tan 0r,rr= 0.022 0.815 GMr,rnt

Kn

0.02700

-8890tr

fJ67

Tan 9,^*."tsr= 0,067.


0.741

0.09042

Suspended Weights (Rev. 03/09/01)

t7

Suspended Weights (Rev. 03t09101)

t8

Answers Prepared by MCA OOW Nuri KAYACAN

Dec 2012

(d)

Calcalate weight of ballast to transfer from Stbd. DB tank to Port DB tank.


The key point here is to firstly identifu the situation during the lifting operation which will create the maximum list. Drawing a
diagram will help.

In theJnal condition G is offhe centreline by 0.022 m (GG*)

GG..:wxd
D-

0.022: wx

w=0.022x 8890

24.5t

8890

Transfer 24.5 t

Consider the following example.

A ship dsplacing 16200 t is uprght and las a 90 t weight on deck at Kg 13.0 m, 6.00 m to port of the centre-line. This weight list s not o exceed

Exsmple

is lo be dscharged into a lighter on lhe port side, 14.00 mfrom the centre-Iine usng lhe ship's heavy lift derrick. If the angle of

8o at any time during the operation, calculate the maximum allowable KG prior lo dsclarge given tharKMs9.60m. (., ' :i l i l'''' ' : li] ..l] Il' i 'i 1'| ,;]

Suspended Weights (Rev. 03/09/01)

t9

Suspended Weights (Rev.

20

03/09/0t)

10 Answers Prepared by MCA OOW Nuri KAYACAN

Dec 2012

Solution Maximum list will occur when the weight is suspended,at the
derrick head and the derrick is plumbed over the port side.

Calculate the distance that G will be off he centre-Iine (GGH) when the shp is at it's maximum angle of list.

,,)

GGn :wxd Maximum allowed

GGH: 90x(14-61 : 0.044m 16200 |


,

lst

is

8'.

GGH iS 0.044 m. This allows the

GM to be calculated.

TanO**usr=
I

GG.n

GMrr*

Tan
I

8":9W

GM,,,
v

Therefore: GMr,u:0.044 = 0.313 m


Tan 8o

minimum GM of 0.313 m is requred when the weight s plumbed over the side at Rg 27.0 m
9.600 m 0.313 m 9.287 m

GM will have it's minimum value.



I

KM

MinmumGM MaxmumKG

'

GH

G
I

Tan Orn*.ror:

Es

GM''n'
Suspended Weights (Rev.
03/09/01)

Suspended Weights (Rev. 03/09/01)

22v

t1

Answers Prepared by MCA OOW Nuri KAYACAN

Dec 2012

Havng calculated the moximum allowed KG when the weight is suspended from the derrick head (Kg 27.0 m), shif he weight back to it's original stowage position on deck (Kg 13.0) to calculate the maximum initial KG requiredfor the operalon.
Take moments about the keel.

': : r.. ,l i-----\ :i i'(' t:;

weisht (t)

Ke(m) moments (t-ml


9.287 27.00 150449.4 -2430.0
t

)j

t6200
-90.0

ton>

ffiE

900

t3 00

170.0

TIe nitial KG must not be greater than 9.209 m in order to limit l"utt'.' 5-r' the list to 8". -] r,^" .': -----. -'.- _, _],-;,),..

,'

Z*J

\''!f

Suspended Weights (Rev. 03/0910t)

23

Answers Prepared by MCA OOW Nuri KAYACANt2

Dec 2012

The following procedures should be carefully observed:

During the course of a voyage the stability of the ship should be closely monitored. It is recommended that a calculation of flud GM and a coTesponding GZ curve be produced fo the wost anticpated condition. Calculations should be done for both departure and anticipated arrival conditions, these being adjusted.to suit any changes that may take place as the voyage
progtesses.

1.

Alter course to pul the ship's heod into lhe predominan wave.

If the ship is in a lolled situation it is essential that the ship stays

An angle;of loll situation may arise in ships carrying timber deck cargoes. Deck cargoes will absorb moisture causing G to risd.'fuel and water will also be consumed from low down in
that may have been initially full. These effects should be
accounted for. Poor tank management may cause excessive free surface moments, leading to a simila loss of stability'
the ship raising G further due to the removal of weight from low down in the ship and the introduction of free surfaces in tanks

lolled to the same side. Wave action may cause the ship to roll through the vertical to loll on the other side. This is a dangerous situation since the ship will heel from the vertical of it's own accord and the momentum it will have in lolling over to the other side may be suffcient to capsize it. In any event, the ship will initially heel beyond the angle of loll before settling at the angle of loll whereby cargo shift may result which will worsen
the situation further.

2.

Check that port and starboard lisng momenS ale


lhe same.

Unfortunately things do not always go to plan and mistakes may be made in calculations. Any number of things might happen such as a collision or a fire where water is introduced into a compartment for fire fighting purposes. Whatever the cause, a situation might arise whereby you find that the ship is lying at an unexpected angle of inclination.

soundings and checking for cargo shift it should be possible to account for any listing moments that may cause the ship to be in a listed situation. If it is calculated that there are no net listing moments then a case of instabili may be assumed and the ship will be lying at an angle of loll.

By verifying tank

3.

Checkfor slack anks

It is not possible to ascertain easily whetler a ship is listed or lolled and since the remedial action for each case is very different it is essential that the cause of the inclination be carefully investi
Correcng an angle

In carrying out (2) above it should also be evident if there are excessive free surface moments causing a loss of GM suffcient enough to make the ship unstable. In this case a loll situation may be confirmed.
ofloll

(MAR Rev. 09110/04)

ofloll

Correcng an angle

(MAR Rev. 09ll0l04,

Answers Prepared by MCA OOW Nuri KAYACAN

Dec 2012

4.

Take aclon to lowe G (reduce

KG)

5.

It would be impractical to consider shifting weights on board using ship,s lifting equipment at sea. If the ship has high ballast tan<i tat are fuil then these may be emptied, dschargng ballast from the high sde tank frst - the greaer verlcal dstancZ between G of he shp and g of he wegh heng dschargedwll ensure tha the greatesl lowerng of G wll take place n hefts nstance-

' sounded all the tanks any that are slack will be Hauing identified. Minimise the loss of GM due to free surface effect by topping up low down ballast tanks and transferring fuel as
necessary. This action alone may remedy the situation'

Minmise Free surfaces

6,

Bollast lanks low down n he shp

once the high side tank is empty the one on the low side may
then be emptied.

select a set of suitably subdivided double bottom tanks to ballast. Ideally start with tanks that have the smallest free surface areas to minimise the effects of free surface whilst filling. The order of filling is as follows and must be strictly
adhered to:

(a)

surfoces whlstJtling the stuation will nitolly

centre tank (No' 1\ as shown. Because of he introduction of more free Start

by filling the

Correcting an angle ofloll (MAR Rev. 09110104)

Correcting a angle ofloll (MAR Rev. 09110104)

Answers Prepared by MCA OOW Nuri KAYACAN

Dec 2012

(b) (c) (d)

When the first tank is completely tank (No. 2t.

full,@.
the final tank (No.

Note

When the low side tank is full,


3)

fill

tme.

once u loll situalon i conftrmed


I

oqly,. ever

JIl

one, tonk..at,-o

'.,

complete in an uprgh condton (having initially . .verified that the port and staoard moments were e
sane).

If G is lowered sufficiently then the ship should

Always JiIl !g",'i4q lonks Jrst (or cen|p tank and then low

side).

The movement of G after completely filling each of the tanks is as shgyn (ignoring the upward movement of G which arises as a result of the introduced free surface at intermediate stages of

ftrst
If
aon

filling the tanks). tan

inqy'*'ume l s a

hqr..

a!y d91b as to whe|her;the,shB


IoIIed

iuni,n aia

G.

G(

c.: f,.G.

low down in the ship) may be discha-rged,


2

To ticqt,q loll silugtion n lht sameway woald,I1ave dsoslro4conseqaences for reasons alrea .fl'toiqed!

If the situation is not remedied then a second set of tanks must


be chosen for flling, the process is repeated.

Correcting an angle of loll

(MAR Rev.

09110104)

Correcting an angle ofloll (MAR Rev. 09110104)

Answers Prepared by MCA OOW Nuri KAYACAN

Dec 2012

i) .'
TUTORIAL OUESTIONS
1.

Calculate the effect on G of shifting 250 t of crgo do,nward through 15 m from a posion on deck to the lower hold in a ship displacing 22000 t.
(0.182 m down)

2.

ship displaces 24300 t and has a on deck at Kg 9.60 m.

KG of

5.60 m. Calculate

e fial KG if

180 t is loaded (5,629 m)

3.

ship displaces 12260 t and has KG 6.68 m. A weight of 34 t is discharged from the lower hold, Kg2.2 m. Calcate the final KG.

(6.6e2 m)

ship has an.inia KG of 6.20 m. If the displacement is 23360 t, what.is the marim:m aont of cargo at may be loaded on deck in a position Kg 10.40 m to ensre that the final KG does not exceed 6.46 m?

5.

ship displaces 12300 t and has KG 5.84 m. What is the marimrm amornt of cargo at can be discharged fron the lower hold, Kg l.80 m, to ensre ttat tte fina KG does not exceed-6.00 m?

(1541.5 t)-

,
I

(468.6 t)

6.

ship displaces 2730 tonnes and has aKG of 6.00 m. The ship then loads the following: 540 torues at 5.0 m above the keel; 370 tonnes at 8.5 m above the keel; 110 tonnes at 10.4 m above the keel; 850 tonnes at 4.6m above the keel. Calculate the'finalKG.

(5.9j0 m)
after the

7.

Ioadetighter displaces 856 tonres and has a following weigbts have been discharged: 160 tonnes from 2.5 m above the keel; 40 tonnes from 3.7 m above e kee[; 395 tonnes from 1.2 m above the keel.

KG of l.5 m. Find e new KG

(1.00a m)

8.

ship leaves port with a splacement of 9060 tonnes and a voyage e following is consmed: oilfuel: 200 tornes from 0.8 m above the keel; 320 tonnes from 0.7 m above the keel; Stores: 98 tonnes from 9.5 m above tte kee[; 87 tonnes from 10.0 m above the keel. FW: What will be the KG on arrival at port of destination?

KG of 5.2 m. During

the

(5.377 m)

KG CALCULATI6NS (Formulae meod

and moments) - Tutorial Questions (Rev. 03/09/02)

- Answers Prepared by MCA OOW Nuri KAYACAN $o

Dec 2012

4285 tonnes, KG 6- O m. The following is loaded: ove the keel; ove the keel; ve e keel.

|^

t0.

"H: ffi fifr' -Load: 50 g5 22 Dischage: ;;

of 6'5 m and a dsd.a9ement


g weights: bove the keel; bove e keel;
bove the

of

Q.a59

n)

6020 tonnes. Find e new KG after

kee

700 toues from

2.6fr;r:fr:I:;

Loadr

worked:

l62m;
.56 m;

,r*.rrr"*:;;;:

('

6 m; Discharge: Shift: 236 tonnes tom r 'o.,l""a

position

on

deck Kg l2.2 m. (6.2s9 m)


as follows:

5.64 m. Cargo is
;

worked

m..

.can-be

loadecl on deck

at

Kg 2.00 m to

ensrre..

()

Q0r.8 t)

j cALcULATIoNs

(,
(Forulae method and noments)
_

Tutorial Questions (Rev. 03/09/02)

Answers Prepared by MCA OOW Nuri KAYACAN

Dec 2012

SOLUTIONS

1. 2.

GGv:wx d w
GGv:
wxd W + r

GGv:250x GGv:
Initiat CGv

22000

16

GGv=0.182mdownward

180 x (9.60 24300 + l80

5.60)

GGv:

0'029 m up

KG (up) FINAL KG

5.600 0.029 5.629

3.
I

GGv:wxcl W-w

GGv:34x(6.68'2'20\
12260-34
6.680 0.012 6.592 6-200 6.460

GGv:0'012mup

/'' 4.
GGv:
wx Initial Fihal

Litia KG GGv (uP) FINAL KG

Mo. GGv

KG KG

(up)

0.260

d W+w

0.260: w x (10.40 - 6..20) 23360+w

Solve the equation for w

0.260 x (23360* w)
6073.6 + 0.260w =

w x (10.40 - 6.20)
6073.6= 4'2v'
_

4.2w
5.840 6.000 0.160 -==
(5.84 -

0'26w

6073.6:3.94w

w = 1541.5 tonnes

\ ).
GGv=wx

Intial KGFinal KG= Mar. GCv (.up)

d W-w

0.160:wx

12300-w

1.80)

Solveeequationforw

0.160 x (12300 - w) 1968 - 0.160w:

:w

x (5.84 - 1'80)
1968 = 4'04w + 0'16w

4.04w

1968:4.20w

w=468.6tonnes

KG CALCULATI9NS (Formulae metlod

and moments) - Tutorial Questions (Rev.

3L KAYACAN Answers Prepared by MCA OOW Nuri

03/09/02)

GF

,-.i

-"':

: i*''

l:f .

..,
'

Dec 2012

6.

Take monents abut e keel.

Itreight (t)
Itrtis Ditpl

Kg
6.00 5.00 E.50
10.40

hilonenb (tm)
I63EO

?734

LOAI} LOAI} LOAI}

5{)
3TO

LoAI
7.

2700 3145
t

44

ENAL

850
4fi{il}

4.60

5.99)

39t0 2.n79

Take moments about the keel.

Take moments'about the keel.

wight
Intil Dsul

(t)

Kg
5.20
0.80 0.70

DISCH DISCH DISCH DISCH

9060 -200 -320 -98


-87

l\ilomrnb (tm) 471l2


\

-160 -224

9.50
10.00.

-931
-870 44927

FINAL

8355

s.377

9.

Take moments aboutthe keel.

Moments (tm)

('
KG CALCULATIONS (Formulae method
ad rnoments)
_

Tutorial Questions (Rev. 03/09/02)

Answers Prepared by MCA OOW Nuri KAYACAN

33*

u.r.,

.-.i r1;. :..

- ,

Dec 2012 '.-.Itu

t.l rli,

Iu.

Take popents aboutthe keel.


I

Weight (t) Initial Displ


6020
500 850

Ks
6.50 2.50
5.00 8.40

Moments (tm)
39130
1250

LOAI) LOAI) LOAI) DISCH DISCH F'INAL- (

4250 r848 -1815 -1820 42843

220 -330 -700

5.50 2.60 6.531

6s60_-

1.

Take moments abouttte keel.

Weight (t)

Kg
6.22-_

Moments (tm)
117806.8:,

LOAI) LOAI) LOAI) LOAI)

tnitial.Dsnl

18940. 200
188

8.62 2.56 3.46 12.20 7.68 4.20

46 236 -562 -236 18812

FINAL
(

DISCH DISCH

6.259

1724 481.28 1s9.16 2879.2 -4316.16 -99t.2\ 117743.08

KG CALclJLATIoNs (Formulae nethod

and moments)

(Rev.03/09/02) Tutorial Questions Answers Prepared by MCA OOW

6 Nuri KAYACAN

3+

Dec 2012 i

L2.

Take moments about the keel, let x = cargo to load on deck at Kg t2.00 m. The fnal KG known which is 5.80 m.

is

7-
\;

Initial Displ

Weght (t) r6420


1500 1200

Ks
5.64 6.50 5.00 4.20 8.20
1.50

Moments (tm)
92608.8

LOAI) LOAI) LOAI) LOAD DISCH


Deck cargo

9750
6000

900 1000 -220

3780 8200 -330

x
20800 +

12.00

FINAL
Finat

lZx

5.800

120008.8 + 12x

KG

Moments' Fina Displacement


120008.8 + l2x= 20800 + x 120008.8

(r

Therefore: 5.80:
5.80 x (20800 *

x):

+12x

20640 + 5.8x: 120008.8 + l2x

120640 - 120008.8

t2x-

5.8x

631.2:6.2x

x = 101.8

(\

KG CALCULATIONS (Formulae method

and rnoments) - Tutoriat Questions (Rev.03/09/02) Answers Prepared by

MCA OOW Nuri KAYACAN

3s*,

Dec 2012 n'?

--ASGOW rlCOLLEGE OF NAUTICAL


I

STTJDIES

KG CALCT]LATIONS
1.

A ship displaces 2730 awes


540 tonnes 370 tones 110 tonnes 850 tones Calculate the fnal

KG.

at 5.0 m above e keel; at 8.5 m above the keel; at 10.4 m above the keel; at 4.6 m above the keel.

and has a

KG of 6.00 m. She then loads the following:

(s.9i

m)

2.

(-

loaded lighter displaces 856 tonnes and has a KG of 1.5 m. Find e new KG after the following weights have been discharged: 160 tonnes from 2.5 m above e keel; 40 tonnes from 3.7 m above the keel; 395 tonnes from- l.2mabove the keel.
(1.00a

n)

3.

ship leaves port with' a displacement of 9060 tonnes and a KG of 5.2 m. Duing.the, voyage the follouing is consmed: 200 tonnes from 0.8 m above the keel; Oil tuel:
I

What uill be the KG

FW:

Stores:

320 tonnes from 0.7 m above the keel; 98 tonnes from 9.5 m above the keel;.. t7 tonnes fiom l0.0 m above e keel. on arrival at port of destination?

,,u,^,

4.

The original displacem.ent of a ship was 4285 tonnes, weights: 800 tonnes at 3.6m above the keel; 440'tonnes'at'1.0 m above the keel; 1 10 tonresat'S,8 m abovete keel; 630 tonnesat.3.O mabove-the keel. Find thenewKG.

KG

6.0 m. She loads the following

(5.459 m)

5.

6.5 m and a displacement of 6020 loading and dischargng the following weights: 500 tonnes at2.5 m above the keel; Load: 850 tones at 5.0 m above the keel; 220 tornes at 8.4 m above e keel; Discharge: 330 tonres from 5.5 m above the keel; 700 tonnes from 2.6 m above e keel;

ship has o

KG of

Find the new KG after

(6.531m)

MAR Rev l2ll2l00

Answers Prepared by MCA OOW Nuri KAYACAN

3LE

Dec 2012

6.

Find e new KG of a lighter which has loaded and discharged e following weights: 140 tonnes from 2.5 m above the keel; Dischaged: 270 tonnes from 1.4 m above the keel; 215 tonnes at l.0 m above e keel. Loaded: and KG were 650 tonnes and 2.0 m respectively. displacement The original
(1.730 m)

(i

7.

The light displacement of a ship is 2875 tonnes. 390 tonnes is loaded 7.0 m above the keel and 7l0 tores at2.5 m above the keel. If the KG was then 5.2 m what was the light KG? (s.622 m)

A ship displaces
worked: Load:

18940 tonnes,

KG 6.22 m. Calculate e finat KG if the following cargo is

Discharge: shift:

200 tonnes at Kg 8.62 m; 188 tonnes at Kg 2.56 m; 46 torues at Kg 3.46 m; 562 torues from Kg 7.68 236 tonnes from Kg 4.20 m to a position on deck

m;

Kg

12.2 m.

(
(6.259 m)

'i

9.

A ship displaces

Load:

16420 tonnes, KG 5.64 m. Cago is worked as follows: 1500 t at Kg 6.50 m; 1200 t at Kg 5,00 m; 900 at Kg 4.20 m; 1000 t at Kg 8.20 m;. Discharge: 220tfromKg 1.50 m. what is the maimrm arornt of cago that can be loaded on deck at Kg 12.00 m to ensrue that the final KG does not exceed 5.80 m?

Q01.8
10.

t)

ship has an inial displacement of 14000 tonnes, follows: 220t at Kg 3.20 m; Load:

KG

5.90 m. Cargo is then worked as

Discharge:
A weight of

Kg 10.60 m. Calculate the maximr height at which a fnal weight of l00 torues may be loaded to ensure that the fna KG does not exceed 5.96 m. Q.61a m)

146 tonnes is then shifted from the lower hold,

86 t at Kg 8.00 m;500 t from Kg 6.20 m.-

(
Kg 2.20 m, to the upper deck,

IAR/I(G
tARRev |2/I2l00

cALcULATIoNs

l'

Answers Prepared by MCA OOW Nuri KAYACAN

3l

Dec 2012

GLASGOW COLLEGE OF NAUTICAL STUDIES

KG. GM & SUSPENDED WEIGHT PROBLEMS INVOLVING WRTICAL COMPONENTS

1.

A ship of 1200 tonnes displacemenq KG 230tat aKg 5.3 m; 420tat aKg 3.3 m;
The

3.1 m,loads cargo as follows:

(a)

240tataKg3.l m. KM after loading is 3.7 m.

Calculate the KG of e ship on completion of cargo.


(3.382

(b) 2.
(

Calculate the final metacentric height.

n)

(0.318 m)

A vessel

has a displacement of 32450 tonnes, KG 8.23 m, KM 8.97 m (constant) and is to quantity of deck cargo Kg 15.0 m. Calculate the quantity of deck cargo that can be' a load the GM sailing will be 0.4 m. loaded so ttat
(17r5.9 t)

3.

Tht folowing information relates to a box-shaped.vessel fioating in dock r,ater. RD-I.025i-. 70 m,B 12 m, Depth 7 m,KG 4.2 m, BM 3.3 m, Draughts (even keel) 3.64 m. ,weight of 76 tonnes, Kg T.4 m is to be lowered vertically to a postion in the lo'ver hold A will be 2.5 m. its Kg where Calculate e GM of the vessel after the weighthas been moved.

LBP

Q.0an)

4.

A vessel has a displacement of

14400 tonnes.

tonnes, Kg 2.0 in is to be moved vertically upwards

KM

8.0 m;

KG 7.28
wi

m.

A heavy lift of

85

and re-stowed in the tween declq

Kg 9.0

mi The vessel's own heavy lift derrick is to be used for the operaon, the keel. the minimum GM; Calculate: (a)

its head 20.0 m above

(b) 5.

(0.614 m) (0.679 m}

the fnal

GM.

A vessel displacing 4500

tonnes has a constantKM 6.2 m and present KG 5.8 m. The vessel Kg 8.5 m and then moves 100 t of cargo from the tween deck (Kg 5.5 m) to the lower hold (Kg 2.3 m), all weights being on the centre line.

loads 50 t of cargo on deck at Calculate tte final

GM.

(0.44 m)

6.

A vessel is to use her ovn derrick to move cargo already on boad,-the derrick head being 25 m above e keel. 8 packing cases, each weighing 5 t are to be moved individually from a
position 3 m above the keel to a position 8 m above the keel. The vessel's present displacement is 5000 tonnes, KM 7.0 m and the least GM Calculate:

(a)

KG

6.2 m.

(b)

e fnal GM

Q.7a3 n)
(0.76 m)

KC, GM & SUSPENDED WEIGHT PROBLEMS INVOLVING VERTICAL COMPONENTS OF SHIFT OF G ONLY (Rcv.
z2toa0B)

Answers Prepared by MCA OOW 3 Nuri KAYACAN

Dec 2012

fl

8,500 tonnes and has a KM of l0.2 m (assume costan$ -T9 1 is to be discharged fiom a position on the cente-line at a K of is 25 m above the keel. , 5.0 m using the vessel's own derrick. The derrick head

A upright vessel displaces KG oi g.s m. A 40 t weight

\i

Calculate the GM when: (a) theweightis liftedjustclearofthetanktop.

(b)
E.

(0.i06

n)

t}e weight is finally. discharged ashore.

(0.377 m)

A vessel displacing 9420 tonnes has a.KM of 9.0 m (assumed and discharged as follo*s: Weights " --q are then loaded_a's.i.
3000

constant) and a

KG of

8.5 m.

Kg 4.5 m loaded;. 1400 t gargo Kg I0.$ m loaded; t cargo 2000 ' !000 t.fuel Kg 2.0 m loaded. Calculate the fnal GM of thovessel (answer to three dcimal places).

tba!J'

dilcharged;

'

(0.s80 m)
)

A vessel is initially displacing 5000 t. KG 7.90 m; KM E.90 m (cons!ant). A 30 t weight is to L" a'"r"'ged from a iosition on the cente_line, Kg 4.5 m, using the vessel's own derick.
The denick head -is 30 m above the keel. Calculate the GM whei:(a) ln" *r*n is lifted just clear of it's initial stowage position;

(b)

(0.847 m)

the weight is fnally discharged ashore.

(0.9795 m)

()

Zq
COMPONENTS OF SHIFT OF G ONLY KC, GM & SUSPENDED WEGIIT PRoBLEMS INVoLVING VERTICAL
2A0U03)
(RCV.

Answers Prepared by MCA OOW Nuri KAYACAN

Dec 2012

GLASGOW COLLEGE
.l

iI

OF

NAUTICAL STUDIES
SHIFT OF G/KG PROBLEMS

.t

SECnoN a: SHIFT o{ G

1.

Calculate the shift of G which results fiom loading 500 tonnes of cargo at a height of 3 m' above the c'entre bf gravi of the ship, if the initial displacementtis 7000 tonnes. Is this a rise a fall in KG? Is it an increase or a decrease in GM?

(0.2 m)

Z. ' A ship displacing 1i000 tonnes has KG 7.15 m and KM 8.0 m before discharging 1000 tonnes of cargo. from Kg 2.15 m. Calculate the new GM. Could the ship sail safely with is '.-lalu-e-bTcM?--- t le.35 n) . 3. A ship displacing 5400 tonnes has a GM of 0.25 m before taking on 600 tonne! of bumkers at
.

. '4.

Kg 0.5 m, when the GM rises to

What must have been the original KG?

0.85 m.

(6.s m)

,Erc SECTIONB:SIMPLEKG ..
5.

A.ship has displac"-.nt 15550 tonnes and GM 0.65 m. 500 tonnes of cargo is transferred'' fromNo.. ZLP^to the foredeck (a vertical rise of 8.25 m): What is the new GM? (0.385 n)

.
KG

.
7.24 m on arival at a port where

A
as

ship has displacement |24oo tonnes

Discharged 1000 tonnes fiom Kg 8.12 m; Discharged 1500 tonnes frorn Kg 4.56 m; T-aaded'2200 torubs at Kg 6.42 m. Calculat e KG on departre.

foliows:

Fgo
\

, worked

(7.3sm)

W1iat,change.in KG,will occuria! a result.of e following,work taking place aboard a ship displacing 12000 tonnes andwithKG of 6.E7 m?=

Loaded 1525 tonnes at Kg 8.23 m; Loaded 876 tonnes at Kg 3.69 m?

(Scn drop)

7.

the following information ' f.. Length of vessel 80 m; Bread

calculate the KG on completion of loading: 1l.7 m; Block coefficient 0.8. Draught of vessgl: 4 m in water of RD 1.024. Initial GM 0:84 m. KM 5.8 m (assume constant). Cargo to load 170 tonnes atKg 6.2 m.

(5.025 m)

8.

From the following information c.alculate the quantity of deck cargo to load so that the vessel will sailwith a.GM of 0.5 m; Displacemnt priorto loading deck cago 22450 tonnes. Initial KG 8.37 m. KM 8.97 m constant. Kg of eck cargo 15.0

m.

(34s.8 0
.:l'

SHIFT OF G/KG PROBLEMS (MAR Rcv' 23.rcA0E)

4o KAYACAN Answers Prepared by MCA OOW Nuri

Dec 2012

g.

From the following inforration calculate e quantity of deck cago to load so that e vessel will sail wi a GM of 0.42 m: Displacement of vessel prior to loading: 4200 tonnes, KG 5.85 m Cargo to load below deck 6200 tonnes Kg 5.95 m. Kg of deck cargo9.7 m. Assme KM 6.5 m constant. (489.s 0 From the following details calculate e GM of e vessel at the time of sailing: Displacement prior to commencing cargo 8000 tonnes; KG 7.0 m. Cargo to discharge: 1000 tones, Kg 5.5 m; Cargo to 900 tonnes, Kg 3.0 m; 600 tonnes, Kg 5.0 m. Cargo to Assume KM7.25 m constant.

1^

10.

load: load:

(0.6a m)
I

l.

From the following details calculate the weight of cargo to load so that the vessel will sail with a GM of 0.35 m: Displacementpriorto loading 1800 tonnes KG t.3 m. Cargo loaded: 800 tonnes at Kg 4.88 m; Space available forremaining cargo-Kg 6.8 m. Assume KM constanttbroughout at 5.0 m.
(207:4 t)

(
I

12.

From the following information: Calculate the GM on completion of loading i prior to loading the displacement was 5400 tonnes KG 4.5 m, cargo-loaded was 550 tonnes at Kg 4.3 m and.-420 tonnes-at: K95.7 m, KM 4.85m and free surface effect is 0.15 m.

(a)

Q.Ia

m)

(b)

Is this a safe

GM? Give

reasons for your answer.

(,

(
SHIFT OF G/KG PROBLEMS (It ARRc,v.23l0t/03)

Answers Prepared by MCA OOW Nuri KAYACAN

Dec 2012

GLASGOW COLLEGE OF NAUTICAL STUDIES SEIFT OF G. KG AND GM


'Single weght'formulae to be sefor each question
Assume KI,I to be constat in each question.

1.
2.

Calculate the effect of tasferring250 tonnes of cargo from deck stowage to lower hold stowage (avetical shift of 16 m) in a ship displacing 20,000 tonnes. (0.20 n do,n)

m. What rise of G

l2O tonne heary lift resting initially at a Kg of 2.5 m is to be restowed on deck at Kg 12.5 will occur as a resul given at the ship's displacement is 24000 tonnes.
(0.05 mp)

J.

(
4.

If the lift in the previous question is to be restored using the ship's heavy denich e head of which is 25 m above the keel, what will be the effect on the GM when the load is lifted just clear of the deck of the hold? (0.1125 mdeuease)

What will be the effect or G of tras'errig8O'torresof cargofrortNo:3'TD'(Kg l0 m;6 m-: to port of CL) to No.3 LH(Kg 2 m;.4 m to starboard of CL) in a ship of displacement 16000. tonnes?
(0.04 m down: 0.05 m to stbd)

5.

A ship displacing 12100 tonnes has KG atKgl2 m. Calculatethe new GM.

7.0 m and

KM

7.8 m. 150 tonnes of cargo is loaded (0.73e

n)

6.

has KG 7.12m and KM 7.79 m while displacing 8250 tonnes. What will bethe metacentic height after 200 tonnes of cargo is loaded at Kg 2.19 m? (0.787 n)

A ship

7.

(
8

Where should the cargo in question 6 have been loaded if the final GM was required to be 75 cms? (K9.3,740 m)=

In a vessel dispJacing 90O0-tonnes'with KG 5.75 m, KM 7.90 m, 500 tonnes of cargo is discharged frorn Kg 2.5 m. What is the final GM? (0.900 n)

9.

A ship displaces

fiom K94.5 m. Findthe GM.

12200 tonnes with

KG

7.22 m and

KM

8.00 m. 700 tonnes ae offJoaded (0.614 m)

10.

displacement 9250 tonnes, KG 7.4 m, KM 8.0 m. From what height above the keel should 500 tonnes be discharged in order to complete with a GM of 70cm?

A ship has

(9.15 m)

4>
Shift of G, KG and GM (MAR Rev. 2UOA03),

Answers Prepared by MCA OOW Nuri KAYACAN

Dec 2012

GLASGOW COLLEGE OF NAUTICAL

i)

STT.'DTJES

KG/GM PROBLEMS INCLUDING FREE SITRFACE EF 'ECT

Calculate the GM of a vessel on arival at the discharging port from the foltowing details: Prior to loading displacement 6050 tonnes KG 5.65 m. Cargo to load 2606 tonnes Kg 4.03 m; Cargo to load I100 tonnes Kg 8.15 m; Bunkers to load 600 tonnes Kg 1.00 m; Stores to load 250 tonnes Kg 9.95 m. Bunkers used on passage 250 tonnes Kg 0.9 m. KM on arrival 7.00 m.
(0.96 m)

Calculate the amount of deck cargo to load in port

aGMof 0.5 m. At port'A', prior to loading deck cargo,


with
consumption on passage:
I 15

'A'

so that the vessel

will arrive

in port

'B'

displacement

is 11800 t KG 7.90 m.

Estimated

tonnes fuel Kg 1.10 155 tonnes fuel Kg 6.60 85 tornes F'WKg 7.88 25 tonnes stores Kg 10.0 Position of deck cargo: Kg 12.80 m Assume KM 8.6 m remains constant.

m; m;

(l

m;
m.

',

(287.5 t)

Assuming at a certain timber cargo will absorb l0% of its own'_weight in moistre during a voyage calculate e amount of timber to load on deck at Kg 8.58 m in port 'A' so at e i vessel will reach port'B' with an effective GM of 0.35m.

Atport'A' priortotoading
ArrivalKM 6.ll
m.

deck cargo, displacement is9275tKG 5.50 Estimated consumption on passage I 12 tonnes fuel and water from Kg 0.75 effectcaused is 0.114 m.

m.

d,

free surface

Qso
4.

t)

Calculate e change,in transverse metacentric height of a box shaped vessel of 15 m breadthi,and a salt water draught of 8 m on passing from wate of relative densi l.025 to water of relative density 1.000. (increase 0.43 m)

5.

vessel KG 4.0 m displacing 8250 tonnes has a righting lever of 0.2 m at 100 heel. On passage a rectangular DB tank containing 120 tonnes of fuel Kg 0.5 m is half consumed resulting in a fiee surface effect of 0.12 m. Calculate e righting moment in the arrival condition for a heel of l0o. (1433.6 t-m)

(i
KG/GM PROBLEMS INCLIJDING FREE suRrAcE ErrpcT ({AR Rcv.
23108/03)

4g

Answers Prepared by MCA OOW Nuri KAYACAN

::'1-,i:ti:.'':' .'
,,

:''.

Dec 2012

:;..

6.

(a) (b)

Show clearly that if a tank is subdivided by a longifudinal bulkhead into two eqal parts the loss of metacentic height due to the free surface effect will be reduced to a quarter of that of the undivided tank.

A vessel of

8000 tonnes displacement in salt water has a double bottom tank eqally subdivided into trree parts by t,o longifudinal bulkheads. The overall dimensions of the tank are l5.0 m long and 21 m in breadth. Find the virtal loss of GM if tle tanks are half full of tuel oil ofRD 0.95.
(0.153

n)

7.

(a)

The effect on stability at sea of free surface in a slack tank is governed by: (a) the vessel's displacement (b) the relative densi of the liquid in e tank. Explain clealy why this is

and

(b)
(

The free surface moment of a double bottom tank for liquids of relative density 1.0 is
350 tonnes-metres.

Calculate the reduction:in the GM of a vessel whose displacement is 2552 tonnes when the tank is partially filled with fuel oil of relative density 0.875.
(0.12

n)

?i:.,:

KG/GM PROBLEMS INCLUDING FREE SLJRFACE EFFECT (MAR Rev. 23108/03)

4+E

Answers Prepared by MCA OOW Nuri KAYACAN

Dec 2012

GTASGOW COLLEGE OF NAUTICAL STTJDIES

ii

KG & GM: SUSPENDED WEIGIITS

l.

A vessel has a displacement of 14400 tonnes. KM 8.0 m, KG 7.28 m. A heary lift of 85 tonnes, Kg 2.0m, is to be moved vertically upwards and restowed in the tween decl Kg 9.0 m. The vessel's own heavy lift derrick is used for e
operation, with its head 20.0 m above the keel. Calculate: the minimum GM; the final GM.

(a) (b)

(0.614

n;

0.679 m)

2.

(a) List the precautions to be taken before discharging a heavy lift using the vessells own derrick.-.

()

O) An upright-vessel-displaces,.8500 tonnes and has a KM of 10.2 m (assume constant) and a KG of 9.8 m. A 40 tonnes weight is to be discharged from' a position on the centreline.at a Kg of 5.0 m, using e vessel's owr denick. The
derrick head is 25.0 m above the keel. Calculate-the GMwhen: e weight is lifted just clear of the tank top; the weight-is finally discharged ashore.

() (ii)

(0.306 m; 0.377 m)

3.

vessel is to use her own derrick to move cargo already on board, the derrick head being 25 m above the keel. 8 packing cases, each weighing 5 tonnes, are to be moved individually from a position 3 m above e keel to a position 8 m above the keel. The vessells present displacement is 5000 tonnes. KG: 6.2 m KM: 7.0 Calculate:,. the least GM; the final GM. (0.743 m; 0.760 m)

(a) (b)

()

4.

A vessel is initially displacing 5,000 t. KG = 7.90 m; KM = 8:90 m (constant). A 30 t weight is to be discharged from a position on the centreline, Kg 4.50 m,

using the vessel's own derrick. The derrick head is 30 m above the keel. Calculate the GM when: the weight is lifted just clear of its initial stowage position; the weight is fnally discharged ashore. (0.847 m; 0.975 m)

(a) (b)

(
MARRev l2JlA00

1
Answers Prepared by MCA OOW Nuri KAYACAN

Dec 2012

ship displaces 9862 \KG 5.32 m and is to dischar ge a364t weight from the lower hold, Kg 2.|2 m, using the ship's heavf lift crare. Calculate the maimum height of the crane head permissible to ensure that the maimum KG during the operation does not exceed 6.00 m.

Q0.5a4 n)

A ship displaces

and is to discharge a220 t weight from the lower hold, Kg 3.00 m, using the ship's heaw lift crae. Calculate the moimum height of the crane head permissible to ensure that the marimum KG during the operation does not exceed 6.64 m. (25.118 m) 16220 t,

KG 6.34 m

7.

A heary lift barge displaces

KG of 6.42 m. A load of 388 t is to be discharged from a position Kg 4.60 m. If the KM for the current condition is
12000 t and has a

7.44 m, calculate the maximum pemissible height of the barge's crane-head to ensure at the GM does not fall below 0.400 m during the lift.
(23.775 m)

TvIAR/KG & GM:SUSPENDED WEIGHTS

MARRev l2ll?J00

Answers Prepared by MCA OOW Nuri KAYACAN

+6

Dec 2012

GLASGOW COLLEGE OF NAUTICAL STTJDIES

t\

1.

A ship displaces 12000 tonnes, KG 230tatKg5.3 m;


420tatKg3.3 m;

CALCULATING GM & SUSPENDED WEIGHTS


3.1 m, and loads as follows:

240taKg3.l m. Given that e KM for final displacement is 3.70 m, calculate: (a) the KG of the ship on compleon of cargo;

(b)

the final metacentric height.

(0.593 m)

Z.

A ship displaces 4500

tonnes, KG 5.8 m. 50 tonnes of cargo is loaded on deck at Kg 8.5 rnand then 100 torrres is moved from the tween deck (Kg 5.5 m) to the lower hold (Kg2.3 m), all weights being on e centre-line. Calcate the final GM assuming that the KIvI is constat at6.2 m. Q.aal m)

3.

A ship

displaces 9420 tonnes,

KG

8.5 m. Weights are loaded and discharged as

follows:

3000 t of ballast from Kg 5'3 m; 1400 t of cago at Kg 4.5 m; 2000 t of cargo at Kg 10.4 m; 10001of fuel atKg 2.0 m. Calculate the final GM if the KM is assumed to be:constant at 9.0 m'

Dischage:

(0:j80 m)

4.

box_shaped vessel has length 70 m, bread !2 m, dep 7 m, KG 4-2 m ard BM of 76 3.3 m. It is floating at an even keel draught of 3.64 m in salt water. A weigbt tonnes, Kg7.4m is to be lowered to a position in the lower hold, Kg 2,5 m. Calculate the GM after the weight has been moved. (1.03e m)

5.

A ship displaces 32450.tonnesr=KG 8.23.m and.KM 8^97 m (constant) and is to-load=ensure that deck iargo at Kg 15.0 m. Calculate the amont of cargo to load on deck to
the sailing GM is 0.40

(1

m.

(I7IS.'

t)

6.

ship initiauy displaces 5000 t, KG 7.90 m, KM 8.90 m (assume constant). A 30 4.5 m, sing tonne weight is to e discharged from a position on the centreJine, Kg keel dring e m above 30 to be is the ship's own derick. If the head of e derick the operation, calculate the GM when: (a) the weight is fted just clea of it's initid stowage position; o the weight is finally discharged ashore'
(0.847m, 0.975 m)

4
Answers Prepared by MCA OOW Nuri KAYACAN

. ,-j'.-..a

]...'jr.1'l 1l

::fi;i

'.:.:tgtk Dec 2012

7.

A ship dilaces 4400

tonnes, KG 7.28 m, KM 8.0 m. A heay lift of 85 tonnes, Kg 2.0 m is to be restowed in the tween deck in a position Kg 9.0 m, The ship's own heavy lift crane is to be used, it's head being 20 m above the keel. Calculate:

(a) (b)
A

ftg minimum \l; the final GM.

(0.614n 0.679n)

8.

ship mst se it's or,n denick to move cargo aleady on board, e head of the derrick being 25 m above e keel dring e operation 8 packing cases, each weighing 5 tones are to be moved indiduatly from a posion Kg 3.0 m to a position Kg 8.0 m. If the ships crrent displacement is 5000 tonnes, KG 6.2 m and

KM

(a) (b)

7.0 m, calculate: the least GM;

e final GM.

(0.743 m,0.760 m)

9.

An upright ship displaces-,8s00 tonnes:and has a K]vt of l0.2 m (assme constant),,= and a of KG 9.8 m. A 40 tonne weight is to be discharged from a position on the cente-line at a Kg of,5.0 m using the,.shiplown*crane*-th_head_of which i25,m-* aboye e keel. Calculate.the_GM -when:= (a), the weight is liftedjustpleaof the-tanktop;
-

(b)

the weight is

finally discharged ashore.

(0.306m,0,377m)

10.

A heavy lift ship displaces.l6040 tonnes

and-has KG'4.22 m and KIvI 5,14 m. A lift of 640 tonnes is to be dischaged using the ship's two heavy lift cranes from a position on deck Kg 4.64 m. Calculate e madmm perrissible height of the heads of both cranes d:ring e lifting operation to ens:re that the GM does not fall below 0.40 m. (1 7.67 m)

MAR/GM & SUSPEI.{DED WEIGHTS.Tutorial

48
Answers Prepared by MCA OOW Nuri KAYACAN

Dec 2012

GLASGOW COLLEGE OF NAUTICAI STIJDIES LIST (OOW LEVEL)


List caused b} a shift ofweght 1. A ship has displacement 3500 t; KM 3.8 m and KG 3.15 m. Calculate the list caused by shifting 15 tonnes athwartships through a distance of 14 m.

2. 3.

(s.3") ship has displacement 6500 t and metacentric height 0.55 m. Find the list produced by shifting 30 t through 6 m transversely in the ship.

(2.88)

A ship of

1000 t displacement has a weight of 5 t moved 4.6 m across her hold. If she is initially upright with a GM of 0.305 m, find the resultant list.

(4.31)

4.

An upright vessel displacing 11850 t has KM 8.4 m and KG 7.3 m. A locomotive weighing 150 t is loaded on deck at Kg 14.8 m, 6.5 m off the centreline. Calculate the
angle of list.

(4.62)

5.

upright vessel has a displacement of 15500 q KG 8.1 m and GM 0.7 m. 500 t of tween deck cargo is discharged from Kg 10.4 m at a distance of 2.4 m from the centreline. Calculate the list caused.

(s.88)

6.

changes in loading then take place: 8030 t of cargo loaded at Kg 6.35 m on the CL; 530 t of bunkers loaded at Kg 7.68 m on the CL; 85 t machinery loaded at Kg 14.lm, 5.8 m to starboard of CL; 80 t ballast loaded at Kg 1.0 m, 3.8 m to port of CL. Calculate tte angle of list and direction-

ship of 4000 t displacement,

KG 7.4 m, KM

7.51 m is upright. The following

A vessel of 15500 t displacement, KG 6.0 m, is listed 3" 50' fnal angle of list if she loads e following: 250 t Kg 6.0 m, 4.0 m to port of CL; 400 t Kg 7 .0 m,5.0 m to stbd. of CL; 600 t Kg 5.0 m, 3.0 m to port of CL; 500 t Kg 9.0 m, 6.0 m to stbd. of CL.
Assume

(1.1" Stbd) to port. Calculate the

KM

7.50 m constant.

(1.3'Stbd)

8-

A vessel of 15000 t displacement loads: 600 t at Kg 4.0 m, 11.0 m to port of CL; 250 tatKg 6.0 m, 8.0 m to stbd. of CL;
and discharges:

KM:6.423

350 t at Kg 8.0 m, 9.0 m to port of CL; 450 t at Kg 5.0 m, 4.0 m to stbd. of CL. Calculate the angle of list if the vessel was initially upright with
m.

KG

5.0 m.

Final

(8.06" Port)

LIST (OOW LEVEL) (MAR Rev. 24/08103

Answers Prepared by MCA OOW Nuri KAYACAN

Dec 2012

9.

ship of displacement 7500 t,

changes occur:

KM

7.25 m,

KG

6.5 m is upright before the following

Cargo Cargo

discharged:

loaded:
KM

600
100

940t,

! Kg 4.4m,3.Zmto stbd. of CL; t Kg 1.8 m, 5.0 m to port of CL;


Kg
4-9 m,1.9 m to stbd. of

CL.

Assume

is constant. Calculate the angle of list when the ship completes cargo.

(5.4. port)

10. A

ship of 8000 t displacement is listed 6o to starboard. Calculate the weight of cargo to load in the port tween deck if Kg of the cargo is to be 8.0 m, the space availablels 5 m to port of the CL, KM is 5.5 m and the KG is initially 4.5m.The ship is to complete loading upright.

(t68.I6
i
t
1.

t)

weight to stow in each space to ensure that the vessel completes loading upright. (271.9 t Port; 328.1 t Stbd.) 12.

A vessel of 4000 t displacement, KM 4.25 m, KG 3.5 m, is listed 3.5o to starboard. If 600 t of cargo are still to be loaded and there is space in No.3 TD (Kg 6.5 m and 5.5 m to port of cL) and in No.4 LH (Kg 3.5 m and 4.0 m to stbd. of cL); calculate the

A vessel of 500 t displacement, KG 3.5 m, KM 4.0 m, is listed 4" to port. If 20 t of cargo is stillto be loaded atKg 5.0 m,4.0 m to port of the CL, calculate howthe remaining cargo of 50 t should be stowed to bring the vessel upright. The available spaces are atKg 3.0 m, 3.0 m to port of CL and at Kg 5.5 m,4.5 m to stbd. of CL.
(17 t Port; 33 t Stbd.)

HEAW LIFT DERNCK


1.

\
2

starboard side to the quay. A load of 60 t stowed on deck at Kg 10.0 m and 6.5 m off the CL to port is to be discharged by the ship's heavy lift derrick, the head of which is 16 m above the keel. The load is to be landed on the quay 11 m out from the cL. Calculate the maximum angle of heel during the operation and the final angle of list after discharge. Assume KM constant. (6.22o Stbd; 2.21" Stbd.)

ship of 10000 t displacemen!

KG

8.0 m,

KM

9.0 m, is upright and berthed

ship of 10000 t displacement, KM 8.1 m (constant) and KG 6.8 m has to load 2 x 50 t loads on deck, Kg 10.5 m, and 7.1 m off the cL to port and starboard. The head of the ship's derrick lifting the loads is 20 m above the keel and has an outreach of 10.5 m. Calculate the maximum angle of list during the operation if the inshore load is placed aboard frst.

(4.1)
3.

A vessel of 4000 t displacement heels 10o when a load of 30 t is lifted from a quay by a ship's derrick. If e GM before lifting the load is 0.75 m, KM is constant at7.75 m
and the derrick head is 20.5 m above the keel, calculate the outreach ofthe derrick

(1s.396 m)
4.

A vessel of

2000 t displacemen! GM 0.6 m, lifts a 15 t piece of machinery with her heavy lift derick from the lower hold (I(e 2.0 m) and lands it in a barge alongside. Kg of the derrick head is 14 m ad the oukeach to the barge is 8.0 m. Find the maximum list.

(6.7)

LIST (ooW LEVEL) (MAR Rev.24/08/03

Answers Prepared by MCA OOW Nuri KAYACAN

Dec 2012

NOTES ON
SHIP STABILITY
FOR PHASE s/CLASS
4

POST/OOW

LO-4
J/'f

ooqft4

fofua 'uJ
PoT

-QY

Gla S gg,H,',g,J,gge

Answers Prepared by MCA OOW Nuri KAYACAN

Dec 2012

LONGITUDINAL STABILITY
Trim
I-s_the

(1)

as

measured

difference in cms between the forward and aft draughts,

resoectivelv.

at the forward and aft

perpendiculars

The ship above has draughts F 2.20 m A 2.68 m. 2.68 The trim of the ship is:
2.20 0.48 m by the stern;

48 cms by the stern.

The ship above has draughts F 2.70 m A 2.32 m. 2.70 The trim of the ship is:
2.32 0.38 m by the head;

38 cms by the head.


Longitudinal Stability (MAR Rev 03/01/01)

Answers Prepared by MCA OOW Nuri KAYACAN

Dec 2012

Loneitudinal centre of eravitv (LCG) ['-ih -posi1ion of the ship's centre of gravity relative to the
length of the Ship, Terqgd
!:ly----'-'-'| -'. ;{ 1t i
1{-'

Lonsitudinal centre of buo ancv (LCB


: :) ( .''r(( 'i I ';; l

''';1r'' ii} '-

Grin

djagrpm.
j

' ' -5i--:

l-

''i

"/^:

Is the position of the ship's centre of buoyancy relative to the length of the ship. Is termed B, in diagrams.
''
1'

'

Longitudinal Metacentre IS the point of intersection of the lines of action of buoyancy force acting through the LCB when the ship is in the initial even keel condition and subsequently trimmed conditions. Is
termed

M, in diagrams.

Loneitudinal Metacentric heieht (GMr)


Is the vertical difference between the centre of gravity and the longitudinal metacentre. Termed GM, in diagrams. Consider the ship shown.

Longitudinal Stability (MAR Rev 03/01/01)

Answers Prepared by MCA OOW Nuri KAYACAN

Dec 2012

Consider a ship initially on even keel. A weight already on board is moved aft through'd' metres. This causes G, to move to Grr.

G,G,,-wxd w

Longitudinal Stability (MAR Rev 03/01/01)

Answers Prepared by MCA OOW Nuri KAYACAN

Dec 2012

Rearranging this

gives:
LCB

GrG,

x \tr/

wxd wxd
t-(

Trimming moment
The ship trims until both again.

: G,Gr X W :
and

LCG

are in the same vertical

Chanee of trim (COT)


Is the difference in cms between the trim in the initial condition and the trim in the final condition.

SAO A ship has


undJinul

the

following initial druughts:

after crgo on bourd is shifted. Calculate the change of trim that has occrred.

draughts:

F F

6.00 5.66

m m

A A

5.86 m 6.20 m

Longitudinal Stability (MAR Rev 03/01/01)

Answers Prepared by MCA OOW Nuri KAYACAN

Dec 2012

Answer

Initiul druughts:

Final draughts: F 5.66 m A 6.20 m:

6.00

mA

5.86

m:

Trim

0.14 m bY

HEAD

Therefore:

Trim - 0.54 m bv STERN Chanse of trim - 0.68 m by STER {

:68

cms by STERJY

Moment to chanse trim bv one centimetre (MCTC This is the trimming moment (w x d) required to change the ship;s trim by exactly 1 cm. It is tabulated in the ship's

hydrost atic particulars and used to determine the change in trim when -cargo is either shiffid, loaded or discharged.

COT (cms)

: fv t d :

MCTC

Trimming moment

MCTC

weight of 150 tonnes is moved uft by a distance of 20 m. If the MCTC for the current draught is 250 t-m determine the Jinat trim of the ship if the initial trim was 0.20 m bY the

SAO

steFn.

Longitudinal Stability (MAR Rev 03/01/01)

Answers Prepared by MCA OOW Nuri KAYACAN

Dec 2012

Answer

COT(cm{:wxd
MCTC
COT

:150x20
250

:12cms

0.120 m 0.200 m by STERIV 0.120 m-further bv the STERIY 0.320 m bv STERN

Initial

trim: COT: FINAL TRIM

Longifudinal centre of flotation (LCF or F) Is the geometric centre of the ship's water-plane area at a particular draught and is the point about which the ship will trim.It' positiqn will change with d1aught.

Longitudinal Stability (MAR Rev 03i01/01)

Answers Prepared by MCA OOW Nuri KAYACAN

Dec 2012

The position of the LCF determines how the change of trim (COT) will be apportioned between the forward and aft
draughts.
ir l |; i\ 1 \,, ,.,.)'

Ship with

LCF amidships

If LCF amidships then:


where:

Ta

: Tf: COT
2

: change of draught aft due to trim; and Tf - change of draught forward due to trim.
Ta

SAO

ship flouts ut draughts F 6.50 m und A 6.80 m. Determine the linal draughts if 25 tonnes is moved 45 m forward given that MCTC is 172.5 t-m and the LCF is umidships.

Longitudinal Stability (MAR Rev 03/01/01)

Answers Prepared by MCA OOW Nuri KAYACAN

Dec 2012

Answer

CoT= wxd
Ta

: 25x45 MCTC 112.5


-

=10cms -0.100m

-:...-,

0.100 -- +/- 0.050 m 2 Weight is moved forward so the ship will trim by the

Tf

HEAD.

Initiul
Tfhn-

draughts F
:,

FINAL

".!

'^;t

.['i

J/F

F6.550m

+ 0.050

6.500

A 6.900
A6.750m
- 0.050

Longitudinal Stability (MAR Rev 03/01/01)

Answers Prepared by MCA OOW Nuri KAYACAN

Dec 2012

Ship with LCF not amidships In-Ihi-S ase ih chane of trlm (CoT) wi|l have to_ be ifurnoiid to i6e foiward and aft draughts according to the position of the LCF within the ship's length.

If the similar triangles are considered then:

a: f andTa*Tf:COT Ta Tf Ta: ,a x
LBP
'' '' '{ '''1f ,'"i

{:'

Therefore:

COT

and

Tf: f x
LBP

COT

n..

|-r:{

,,
F

a'

SAO

10.25 m and A 10.15 m. A weight of 95 tonnes is moved aft through a distance of 42 m. Culculate the Jnal druughts given that LBP is 100 m, LCF is 48 mfoap und MCTC is 285 t-m-

ship hus initial druughts

Longitudinal Stability (MAR Rev 03/01/01)

9
{

Answers Prepared by MCA OOW Nuri KAYACAN

Dec 2012

Answer

COT=

MCTC 285 Tu: 48 x 74 : 6.7 cms - 0.067 m


100 TT=

wxd:95x42
7.3

:74cms

52 x 74 =

100 Weight is moved aft so the ship wiII trim by the STERN.

cms - 0.073 m

Initial draughts F

Trim FIIYAL

10.250 - 0. 073 F70.177m

A 10.150
A10.217m
+ 0.067

Longitudinal Stability (MAR Rev 03/01i01)

10

Answers Prepared by MCA OOW Nuri KAYACAN

Dec 2012

The effect of loadine and dischareine weishts

T.

effect of bodily inkuge/rise must be taken into account:

Sinkage/Rise cms

_w

TPC

The following procedure should be followed: 1. Loadldischarge the weight from the LCF, calculating the sinkage/rise using the TPC value given. 2. Calculate the COT by moving the weight from the LCF position to it's actual loaded/discharged position. 3. Find TalTf by apportioning the COT according to the position of the LCF. 4. Apply both the sinkage/rise and TalTf to the initial draughts to determine the final draughts.

SAO

A ship
foap.

100 m in length floats ut draughts F 7.00 m und A 6.80 m. Calculate the final druughts if 150 t is louded 20 m foup given that TPC is 15 und MCTC is 150 t-m and LCF is 45 m

TIP
Alwuys draw a sketch to help you picture what is happening!
Longitudinal Stability (MAR Rev 03/01/01)
11

Answers Prepared by MCA OOW Nuri KAYACAN

Dec 2012

45m
150 t

Answer

Snkage:

w TPC

150 : 10 cms - 0.100 m


15

COT: wxd

MCTC
x 25

_IS0x(45_201 :25cms
150
11.25 cms 13.75

Ta:45 x 25:
Tf

: 55

100

0.113 m

100 Weight is louded aft of the STER]Y.

cms - 0.137 m

LCF

so the ship wiII trim by the

Initial druughts
Sinkage Trim

F 7.000
+
7.1

0.100 0.137
00

FIIYAL

F6.963m

A7.013m
T2

6.800 + 0.100 6.900 + 0.113

Longitudinal Stability (MAR Rev 03/01/01)

Answers Prepared by MCA OOW Nuri KAYACAN

Dec 2012

Loadine/dischareine multiple weishts A tabular approach needs to be adopted where moments are taken about the LCF.
Consider the following example:

A ship 120 m in length floats at druughts F 6.24 m and A m. LCF is 54 mfoap, TPC 14.2 and MCTC 116 t-m.
following crgo is worked: Load 120 t lcg 10.0 mfoup; Load 68 t lcg 86 m foap; Dischurge 36 t lcg 22 mfoup; Dischurge 48 t lcg 60 m foap. Calculate the Jnal draughts.
The

6.36

Longitudinal Stability (MAR Rev 03/01/01)

13

Answers Prepared by MCA OOW Nuri KAYACAN

Dec 2012

TRM - MULTPLE
Enter data

LBP = LCF = TPC MCTC

120 54
14.2

m foap
t-m

116

Weight

(t)

Dist from LCF

Sinkage Sinkage

= g

TPC
COT

= 0.073

7.3

Trimming moment MCTC cms

COT

19.3

Ta
Tf

= =

10.6

8.7

cms

= 0.106

6.313
-0.106

Longitudinal Stability (MAR Rev 03/01/01)

t4

Answers Prepared by MCA OOW Nuri KAYACAN

Dec 2012

Most trim problems are straight forward provided that you


understand the information that is being given and can recognise the formula to which it belongs.

Sinkage/Rise cms

TPC
Trimmingmoment

COTcms: wxd -

MCTC

MCTC

Apportion COT to forward and aft draughts using:

Ta: a x COT and Tf: f x LBP LBP


NOTE

COT

In practice, the mean value of TPC must be used to determine the sinkage/rise of the ship. Similarly, the men values of MCTC and LCF must be used when calculating the change of trim. The change of trim is then apportioned to the final waterline using the Jinul LCF. If a 'hydrostatic particulqrs' table is not given, then it has to be assumed that the values of TPC, MCTC and LCF position do not significantly change i.e.
they remain constant for the range of draughts concerned.

Longitudinal Stability (MAR Rev 03/01/01)

15

Answers Prepared by MCA OOW Nuri KAYACAN

Dec 2012

GLASGOW COLLEGE OF NAIJTICAL STI.JDIES

A stip's displacement scale is usually drawn up for even keel conditions. If the ship has a rcentre of flotation otidships, such a displacement scale will always give the correct displacernent when referred to with e aridshiPs mean draght, regardless of the ship's trim. However, the centre of flotation is seldom aridships. When this is the case, the displacement given by the scale for the (amidships) mean draght will only be correct if the ship is on an even keel. If the ship is trimmed e'amidshiPs meao draght is not a true
measure of displacement on the scde.

TEE TRIIE MEAN DRAUGET : DISPI,ACEMENT WEEN OUT OF TRIM

The sketch shows a ship on an even keel waterline WL for which the displacement is W tonnes. F is abaft of amidships. If a weight is moved aft, without change of displacement, the ship rotates about F to some new waterline such as W2L2. The draught amidships changes (reduces in the case shown). If the displacement scale is now referred to with the ne, atidships mean draught the value obtained wil be the displacement up to the even keel waterline WL. This is less than the true displace,rent CD bv the weight of the layer between WL and WL. The draght at F (if referred to the displacement scale) will ve the correct displacement nd for this reason the draght at F is termed the Te Mean Draaght @,D).

w,
V,
}J3,

r
either'.

l--e-J
o(

t,
Tr^o

tr

r"r,ye-F,r

Ev

KEEL

.L 3

l
*

L2

The displacement scale should be referred to with the draught at F and this draught is

i) ii) ie.

draught forward + trim between forward end and F, or; draught amidships + trim between amidsiips anti F. True mean &oght @,ID)

Te mean

: daughtfwd + (J_ x Trim) L : &aught @,ID) fuaugh amidships + ( d x Trim) L

or;

Answers Prepared by MCA OOW Nuri KAYACAN

Dec 2012

Alternatively, the true displacement is the displacement given by the scalg if entered with the amidships mean dragfrt,plus the weight ofthe layer between WL and w3L3 (shown shaded). This is termed the layer conection The displacerent of the layer correction is giver by:

w(tonnes)

:xx
:
L

TPC

(-d- x

Tim) x TPC

The true mean draught (at F) is greater than the arithmetic mean draught (at aridships) if (i) F is abaft of amidships and the ship is trimmed by the stern or () F is forward of amidships and the ship is trimmed by the head; and layer corrections are then additive (see sketches below).

! cogcr

.(.

These layer corrections (shown shaded) ueadditive to the mean draught displacement. The true mean draught (at F) is /ess than the arithmetic mean (at amidship$ if (i) F is abaft of amidships and the ship is trimmed by head, or () F is forward of amidships and the ship is trimmed by ster, and layer coTedions are then sbtractive (see sketches below).

*
These layer corrections (shown shaded) are subtracfve from the mean draught displacement.

MAR\TMD.DOC

Answers Prepared by MCA OOW Nuri KAYACAN

Dec 2012

'l

GLASGOW COLLEGE OF NAUTICAL STUDIES

l.

TRIJE MEA_I DRAUGHT AND DISPLACEMENT A ship length LBP l22 m has a displacement 10000 tonnes on an even kee draght 7.62 n TPc l9.7, LCF is 4.6 m abaft aridships. Find the displacement
at a draught 7.01 m Fwd 8.23 m Aft.

(Ans 10090.6 t)

2.

ship length l50 ', TTc 20, LCF 5m aft of amidships is at a draught 6.80 m forward and 8.00 m aft. Calclate the quantity to load to put the ship on an even keel draught of 7.70 m. (fuis 520 tonnes)

. 3. .

Estirrate the displacement and weight of cargo on board a ship LBP 150 r' LCF 2 m abaft of amidships at dratghts 6.50 m F and 9.80 m A when there are 300 tonnes of stores, fuel etc. on board, given the following extracts fiom hydrostac data:

(Light) z.so
4.

Draght 8.40 8.00

Displacement

1090
10570

3700 (Ans Displ. 10885t; Cargo 6886t)

A ship has a loaded SW displacement 12000 t at a SW draught 8.54 m even keel. LBP l22 rr, T?c 18.23, LCF l.5m abaft amidships. At present the ship is in dock water density l0l5 km3 at draughts 7.27 mFwd and 9.30 m Aft. Find the
quantity to load to put ship to the loaded even keel draught.

(Ans 536.4 tonnes)

5.

A ship length 120 m designed to float at a trim 0.60m by the stern is floating at draughts 4.2O mforward and 6.00 m aft. The displacement from the scale for a mean draught 5.00 m is 5800 tonnes. TPC 12, LCF '4 m abaft amidships.
Estimate the displacement.

(As 5968 tonnes)

MAR\TMD.DOC

Answers Prepared by MCA OOW Nuri KAYACAN

Dec 2012

GLASGOW COLLEGE OF NAUTICAL STTJDIES


LONGITIJDINAL STABILITY
(1): TRIIVI

TTIIORIAL SffiET I

1. 2. 3.

Calculate e change of trim which results when a weight of 50 tonnes is tarsfered 20 m forward aboard a ship with MCTC 200 t-m. (Scn by lnd)

A ship has a trim of l0 cm by the head before bansferring 200 tonnes of oil from No.2 DB to No.4 DB (a distance of 52 m). Calculate the new trim, given MCTC lt9.l t-m. (4Scmbystem)

A ship

floats at draughts F 6.84 m and A 7.l4 m. Calculate the final trim and fnal draughs after a weight of 42 tonnes is moved a distance of 60 m fonad, given MCTC l05 t_m and LCF is amidships.
(6cm by stem;

6.96 m,

7.02 m)

4.

fransfened.

A vessel has draughts F 6.90 m and A 7.00 m, MCTC l80 t-m. After transfening fiel oil 60 m further aft the draughts became F 6.76 m and A 7.14 m. Calculate the weight of oil
(84 tonnes)

5. 5. 6.

A ship

has MCTC 250 t_m. What weight would have to be moved rough a distance of 20 m in order to change the trim by 30 cm?
(375 tonnes)

A ship

is on even keel at a draught of 7.32 m. what distance must a weight of 120 tonnes be moved if the ship is to be trimmed 15 cm by the stern? Given MCTC 240 t-m.
(30 m)

floats at draughts F |2.24 m and A l2.l8 m. Calculate the weight of fuel to transfer from No. I DB (lcg 182.5 m foap) to No. 7 DB (1c926.5 m foap) in order to achieve a final trim of 20 cm by the stem. Determine also the final draughts. Given LCF amidships and MCTC 200 t-m. (33.3 onnes; F ]2.11 m' A I2.3I m)

e'inp

8.

Prior to sailing a ship has draughts F 9.92 m and A 10.87 m. Calculate the weight of fuel to transfer from No. 7 DB to the settling tanks (a distance of 52.4 m) to reduce the trim to 32 cm by the stern and determine the resulting draughts if the MCTC is 227.5 t-m and LCF is amidships.
(273.5 onneg;

10.235 m,

10.555 m)

g.

A ship floats at draughts F 10.25 m and A 0.l5 m. A weight of 95 tonnes is tansferred aft a distance of 42 m. Calculate the resulting draughts if LBP 100 m, LCF is 48 m foap and MCTC is 285 t-m.

(F

10.177 m,

10.217 m)

10.

A ship completes cargo with draughs F

12.24 m and A 13.24 m. To reduce the trim to 0.5 m quantity a of fuel is to be transferred forward a distace of E0 prior departure to the stern by m. MCTC 400 t-m, LBP is 200 m and LCF is 95 m foap. Calculate the quanti to transfer. Determine the final sailing draughts.

(a) (b)

(250 tonnes;

12.50 m,

13.00 m)

Answers Prepared by MCA OOW Nuri KAYACAN

Dec 2012

11.

A ship

has draughts F 6.66 m and A7 .44 m. it is required to cross a shoal vhere the depth at high water is 7.60 m with an underkeel clearance of 0.3 m. What quanti of fuel soud be transfened through a distance of 32 m in order to achieve the required reduction in draught

aff? Find also the final forward draught given LBp E0 m, LCF 3E m foap and MCTC 96 t-m.

(88.4 tonnes; 6.815 m)

t2.

A vessel

has draughts F 12.00 m and A 12.80 m. On passage 540 tonnes of fuel oil are consumed from tanks 120 m abaft of amidships. Calculate the arrival draughts assuming LCF amidships, TPC 50 and MCTC 840 t-m. (F 12.278

nA

12.306

n)

13.

A ship floats

and

Calculate the draughts on completion of loading given LBP 108 m, TPC t6, MCTC

at draughts F 4.30 m and A 4.80 m. The following cargo is loaded: 55 68 m foap; 100 38 m foap.

tonnes tonnes

LCF

52 m foap.

l8 t-m
m)

(F 4.374

m,

A 4.918

t4.

Prior to working cargo a vessel floats at draughts F 5.00 m and


cargo is worked:

5.10 m. The following

Load 300 Load 150 Load 200 Discharge calculate the new

tonnes 40 m forward of amidships; tonnes l0 m forward of amidships; tonnes 50 m abaft of amidships; 400 tonnes 20 m forward of amidships. draughts given LCF amidships, TPC 25 and

MCTC zzs t-m.

(F 5.00 m, A 5.30 m)

15.

From the following information calculate the draughs fore and aft when 50 tonnes of fuel is transferred aft through a distance of 67 m. Initial draughts F 3.01 m and A3.74 m. LBP I l0 m, LCF 53 m foap and MCTC 67 t-m.

r 2.75
16.

m,

A 3.98

m)

A vessel has the following characteristics: Draughts F4.05 m and A 4.60 m. LBP 60 m. LCF 28 m foap. TPC 7. MCTC 2l t-m. The fore peak tank is to be filled with 35 tonnes of water. Calculate the final draughts after filling the fore peak tank assuming that the tank's lcg is on the FP. (F 4.38 m, A 4.40 m) A vessel floating at draughts F 4.10 m and A 4.85 m has the following characteristics: LBP 100 m. LCF 45 m foap. TPC 12. MCTC 40 t-m.
100 tonnes 75 m foap; 50 tonnes 20 m foap. Calculate the draughts forward and aft on completion

17.

Cargo to

load:

of loading.

(F 4.47 m, A 4.78 m)

8.

A vessel is floating and MCTC 20 t-m.

at draughts F 2.89 m and

4.36 m.

LBP

56 m,

LCF

26 m foap, TPC 6

The following cargo is loaded: 308 tonnes 46 m foap; Calculate the final draughts on

completion.

200tonnes7mfoap.

4.37 m,

A 4.66

m)

Answers Prepared by MCA OOW Nuri KAYACAN

Dec 2012

19.

From the following information calculate the final draughts on completion of loading: Initial draughts F 6.20 m and A 6.00 m. 100 tonnes 120 m foap; Cargo to load:

LBP

150 m. Assume that

LCF 80 m foap. TPC 25. MCTC 120 t-m. TPC and MCTC remain constant over e

50 tonnes 30 m foap.

range of draughts involved. 6.16 m A 6.25

n)

20.

From the following information calculate the final draughts on completion of discharge: Initial draughts F 5.35 m and A 6.24 m. LBP I l0 m. Cago to discharge: 450 tonnes 75 m foap; 5 l0 tonnes 40 m foap. LcF 60 m foap. TPc 2 (average). MCTC 48 t-m (average). F 4.Eg m, A 5.05 m)

21.

A vessel floating

at draughts F 7.25 m and A 8.45 m in salt water has an after Peak tank wi it's lcg on the AP. The lcg of the fore peak tank is 165 m from that of the after peak ank. The vessel must cross a bar with a depth of 8.50 m while maintaining an underkeel clearace of 0.5 m. The following hydrostatic data applies: MCTC 200 t-m.LBP 175 m. LCF 85 m foap.

Calculate: The weight of water ballast to transfer from the after peak to the bar to be crossed with the required cleaance. The final draughts forward and

(a) (b)

fore peak to enable

aft'

(I12.3

tonnes; F 7.73 m, A g.00 m)

1',)

From the given information calculate: the weight of water to transfer from the after peak to the fore peak tank to enable a vessel to cross a bar with the maximum under keel clearance. Bar depth 9.25 m. the clearance overthe bar. Initial draughts F E.45 m and A 8.90 m. Lcg of fore Peak tank is l70 m forward of e AP. Lcg of after peak tank is 5.0 m aft of the AP. MCTC 210 t-m. Bar depth 9.25 m. LCF amidships.

(a) (b) j

(54.0 tonnes; 0.575 m)

23.

From the data given calculate the quanti of cargo to discharge and e draughts forward and aft for dry-docking. LBPI60 m.Initial drafu F 6.25 m and A 6.75 m. Space available for discharge: No. 2 hold, lcg 130 m foap. LCF amidships. TPC 24.44. MCTC I l0 t-m. Trim required for dry{ocking is 1.5 m by the stem. Assume that TPC and MCTC remain constant over the range of draughts involved. (220 tonnes; F 5.66 m, A7.16 m)

24.

A ship 140 m long arives offa port with draughts F 5.70 m and A 6.30 m. LCF is 67 m foap. TPC 30. MCTC 420 t-m. It is required to reduce the draft aft to 6.0 m by running water into the fore peak tank (lcg 67 m forward of amidships). Find the minimum amount of water to
load and also give the final draught forward. (215.4 tonnes,5.96 m)

25.

From the following information calculate the draughts fore and aft when the fore peak is filled with 35 tonnes of water: Initial draughts F 4.05 m and A 4.60 m. LBP60 m. Fore peak ank lcg 58 m foap. LCF 28 m foap. TPC 7.0. MCTC 2l m. F 4.37 m, A 4.42 m)

Answers Prepared by MCA OOW Nuri KAYACAN

Dec 2012

Glasgow College of Nautical Studies School of Nautical Studies

sTABlLlw

TRM - TUToRAL SHEET

-2

1.

From the nformation tabulated calculate the draughts fore and aft when cargo has been completed:
lnitial draughts: forward 2.89 m, aft 4.36 m Length BP: 56m Cargo to load: 308 tonnes 46m forward of AP Cargo to load: 200 tonnes 7m fonyard of AP LCF: 26 m fonvard of AP TPC: 6 MCTC: 20

2.

From the information tabulated calculate the final draughts fore and aft afier completion of loading:
lnitial draughts: 4.10m for'd 4.85m aft Cargo loaded: 100 tonnes 75m fo/d of AP Cargo loaded: 50 tonnes 20m for'd of AP Length BP: 100 m

TPC:

MCTC: 40 LCF: 45 m from AP

12

3.

From the information tabulated calculate the draughts fore and aft when the fore peak is filled with 35 tonnes of water: lnitial draughts: forward 4.05m, aft 4.60m Length BP: 60m Fore peak tank: cg 58m forward of AP LCF: 28m fonnard of AP
21

TPC: 7 MCTC:

4.

From the following information calculate:

a. b.

The amount of cargo to load to bring the ship to an even keel draught The final draughts forward and aft
Length BP: 160m lntial draughts: Forward 6.25m Aft 7.05m Space available for cargo: No. 2 hatch Cg 125m forward of CF: amidships TPC: 24.44

AP

MCTC:

110

STAB t Tutorial/MR/AC

126 -09.02

Answers Prepared by MCA OOW Nuri KAYACAN

Dec 2012

G_AsGoW CoLLEGE oF NAUTCAL STUDIES TRM US|NG HYDRoSTATC DATA TUToRAL 3


'
Use the hydostatic particulars data sheet to answerthese guesfbns

1.

A shp with LBP 1M m arrives in port on an even keet draught of 5.30 m. The
followng cargo is then worked: Discharge 1560 t from lcg 81 m foap; 1700 t at lcg 112 m foap; 2100 t at lcg 41 m foap. Calculate the final draughts forward and aft.

Load Load

(F 6.194 m, A 6.395 m)

2.

A ship LBP 148 m has draughts F 3.92 m and A4.26 m. The following cargo s
worked:
216 t from 1c422 m foap; 400 t at lcg 40 m foap; 600 t at lcg 52 m foap. CalcuIate the final draughts fonard and aft.

Discharge

Load Load

(F 3.874, A 4.985 m)

3.

A ship LBP 148 m has draughts F 5.22 m and A 6.00 m. The following cargo is
worked:

Load

".Calculate the final draughts fonard and aft.

.-n.

Discharge Discharge Discharge

652 t at Kg 6.50 m, lcg 24 m foap; 194 t from Kg 6.4, Jcg 22 m foap; 362 t from Kg 2.8 m, lcg 130 m foap; 145 tfrom Kg 8.9 m' lcg 88 m foap.

(F 3.736 m, A 7.246 m)

4.

A ship arrives in port on an even keel draught of 5.90 m in salt water. LBP is 150 m. A weighl o 224 t is to be loaded in order that the ship sails with a trim of 0.50 m by the stem in salt water. Calculate the position foap to load the weight. Determine the final draughts. (30.38 m foap; F 5.728 m A 6.228 m)

(a) (b)

5.

At the start of a cargo, a ship is floating on an even keel draught of 6.20 m in salt water LBP 140m: The following cargo operations are planned: Load 1800 t at lcg 107.0 m foap; Load 1500 t at lca 42.0 m foap; Discharge 1640 t from72.O m foap. Calculate the anticipated draughts in SW on completion of cargo operations. (F 7.627 m A 6.276 m)

TRIM USING HYDROSTATIC DATA (MAR Rev. '14111/02)

Answers Prepared by MCA OOW Nuri KAYACAN

Dec 2012

6.

A ship anives in

port with draughts F 5.20 m A 5.80 m in SW. LBP 137.6 m. worked as follows: Cargo is Discharge 1350 t from lcA 90.0 m foap; Discharge 800 t from lcg 70.0 m foap; Discharge 720 t from lcg 32.0 m foap; Load 1050 t at 90.0 m foap; Load 700 t at 70.5 m foap; Load 620 t at 28.5 m foap. Calculate the final displacement and draughts on completion of cargo. (10680 t; F 4.826 m A 5.725 m)

7.

A ship LBP 140 m has a

summer load displacement o 14115 tonnes. ln its current partly loaded condition the ship floats at draughts F 5.26 m A 5.48 m in SW. The ship is to complete loading at the summer displacement with a trim of 0.50 m by the stern. The remaining cargo is to be loaded into two holds: lcg 116.0mfoap; No. t lcg 32.0m foap. No. 4 Calculate each of the following: the quantity to load in each of the holds; the final draughts in SW. (1331.2 t in No. 1, 1887.7 t in No. 4, F 6.541 m A7.041 m)

hold hold

(a) (b)

8.

Aship is

floating at draughts F 4.600 m A5.460 m in SW. Atotal oT72to cargo is to be loaded in a position to keep the draught aft constant. LBP 146 m. Calculate each of the following: the distance from AP to load the cargo; the final draught forward. (84.45 m foap; 5.340 m)

(a) (b)

TR|M USNG HYDRosTATlc DATA (MAR Rev. 14/11/02)

Answers Prepared by MCA OOW Nuri KAYACAN

Dec 2012

GI.A*S@W COLLEGE OF NAUTICAL STTJDIES

TRIM MEASI.JRED BY TIIE DIFFERENCE IN POSITIONS OF IJCB AND LCG For a ship to be in longindinal equilibrim LCB and LCG mst be in the same longittdinal posion. If there is a change in e longitdinal distribution of weight a new posion of LCG mst arise; this position can be fond by taking momen8 of weigbt (longifudinally) about ay closen datm _ possibly e aft peqpendiolar. The Eimning moment rhich aises can be measrEd in terms of e separatio betwecn LCG and LCB.
BF

( }' ?oaJ
Yt vl
I

B
F Q')
e1

-(.t

oQ< l AL

Bs,' (rt

/ -/

L,

hlt

The sketch sho,s a ship, dilacing W toues, originay on an even keel (wL), with centres of buoyacy and gravity at B and G respectively ie. in e same vertical line. If a weight is now moved aft. G moYes aft to G1 as shcin. tonnes-metres. This causes e ship to trim by e stern, tipping aod F on to ,aterline WL (and B will then move aft to become vertically below G1 hen a equilibrir state will again be achieved).
The change of trim from e even keel condition car be found as follo,s;

A trimrig couple then exists, e

moment of ,hich

is (w X GG) or (l x BG)

Change of trim = moment = W x BG ,

MCTC MCTC

ofi

w(LcB-LcG} MCTC
fom

It is important to realise that we arc trying to find the change of trim at occurs
the even teeJ

condition.

Answers Prepared by MCA OOW Nuri KAYACAN

Dec 2012

ship dispacing 10000 tonnes on an even kee draught 7.60 metr,es has LcF amidships and LCB 2 m forward of amidships. Shifting a weight aft resutts in LCG being 1.5m forrard of amidships. McTc = 250 tonnes-metts. Calculate e fnal
draughts.

Examole

t,

Trimming moment = W(LCB'LCG) = 10000 x 0.5 = 5)0 tones-metcs Change of trim = moment = 50) = 20 cm

MCTC
F

?50

bt

LcF

is amidships,

Tr=T.=10cn
Final draught = 7.50 m,

= 7.70 m.

Example 2 A ship leng t20 a, arrives in port at draghb F 5.00 m A 5.20 m. The hydrostatic daa for adraght 5.10 m (even keel) at: W = 7000 tonnes, LcF amidships, LCts 1.0 m for,ard of amidships ad McTc 1o0 t-m. The ship no loa.ls 1000 tones, LcG 50 m forward of AP. The daa for a dilacement of 8000 tonnes a: Draqght 6.20 m, LcF amidships, LCB 0.5 m forward of anidships, MCTC 110 t-m. Calculate e final draughs. Becase tle vessel s ntally 20 cm by tle stern tle even kcel postbn of LCB.
Change of trim =

LCG

mst ntblty be abd1t tle

W(LCG-LCB) MCTC
=

20 = 7000(LCB-LCG) 00

(LCB-LCG)

2000 = 0-857 m 7000

0.7143 m for'ard of amidships. Takig moments about aft perpendiolan

LCG mst originally

wt

70m 1m0
8000

LCG =

474998 = 59.375 mfoap 8000

Answers Prepared by MCA OOW Nuri KAYACAN

Dec 2012

but for loaded shiP

LCB = 60.5 m foap (assning (LCB-LCG) = 1.15 n


s abaft

even keel)

Becase

LCG

LCts trim is by e stE.

Chage of trim (from even keel) =

wtlcEl'c(},
MCTC

EO@-EJJa

= 8l.82 cm by sem'

Equivaleot even keel draght is ven as 6.20 m ond F = 6.20-0.fi9 = 5.791m3 A = 6.20 + 0.409 = 6.609 m.

LcF

s amidqhiPs,

Answers Prepared by MCA OOW Nuri KAYACAN

Dec 2012

GT.ASGOW COLLEGE OF NAI..ITCAL STTJDIES

The hydrostatic partiolars for a ship LBP 35 m ae:

CALCULATION OF DRAUGHTS BY CONSIDERING LCB A}ID

IfG

Drauet(d Dil(il

8.1 8.4 8.7

fBC tilt4 22 t6072 ?2.2 L6735 22.4

McTc 180 88 tn

LCF(m) from amidshiPs from amidshiPs

LCB(m)

0.58F 0.47F 0.36F

2.05A

z.n^

2.52A

Estimate e dilacement and position of LCG when draughts ae7.95 for'ad and 9.05 aft. (16336 tonne, 0.85 m aft of aridships)

2.

A ip at dilacemet|n$ t is on even keel. I,CB = 3.70 abat of amidships. LBP = 156 m.

t-8.

She loads 3300 t lcg 3 m for,ad of aridships and 180 t|cg20 m abft amidships. For dilaceme^tl6220 t the equivalent even keel draught is 7.30 m, LCB = 2.25 m abaft amidships, LCF = 5.70 m abaft amidships and MCTC 250

Calculate e fial dnughts.

(F 7.207

m A 7.380

m)

3.

A ship leng.140 m has sw draughts 7.35 m forward and 7.65 m aft. Hydostac data for a mean draught 7.50 m ae:

Calculate the draughts afrcr discharging 3500 toues of cargo lcgII.2 m forrard of AP and 3000 torne lcg 35 m for,ard of AP. Hydrostatic paaiolas for a displacement 6500 tonne are: Draught 3.7 m, LcB 72.20 m forqard of AP, McTc = 110 toue-metp, LCF = 71m forward of AP. (Note to soluo: original separation BG = 0.346 m origina LCG = 7t.034 rn forward of AP Fina LCG = 65.607 m foruad of AP Final separation BG = 5.593 m Fina Trim 389.6 m by stern) (F 1.780 m A 5-676 m)

LCB 71.38 m fonvard of AP, LCF 70.0 m forc/ard of AP.

McTc

Displacement 13000 tonne,


150 tonne-metre,

N{AR\srAB33.Doc

Answers Prepared by MCA OOW Nuri KAYACAN

Dec 2012

G-AsGow CoLLEGE oF {AUTCAL sTUDEs

1.

A ship LBP 146 m anives in

port with an even keel draught of 5.40 m. Cargo is worked as follows: Load 200 t at lcg 42 m foap; Discharge 50 t from lcg 53 m foap; Load 320 t at lcg 100 m foap; Load 197 t at lca 60 m. Calculate the final draughts. (F 5.819 m, A 5.595 m)

2.

A ship LBP 142 m has an even keel draught of 4.20 m. The following cargo is
loaded:

262tat lcg 36 m foap; 304 t at lq 122 m foap; 24tal lcg 140 m foap. Calculate: the final draughts; the ballast to transfer between the fore peak tank and the aft Peak tank through a distance of 134.6 m to reduce the draught to a minimum. (F 4.769 m, A 4-189 m; 66.4 t)

(a) (b)

3.

A ship LBP 152

the final draughts if cargo is loaded as follows: 460 t at lcg 40 m foap; 300 t at lcg 92 m foap; 63 t at lcg 76 m foap.

m floats in

SW at draughts F 4.70 m and A 5.30 m. Calculate

(F 4.882 m, A 5.830 m)

4.

A ship LBP 143 m has draughts in SW of F 5.28 m and A 4.42 m. Cargo is

worked as follows: Discharge 200 t from Icg 103.6 m foap; Load 484 t at lca 60.6 m foaP; Load 73 t at lcg 96.4 m. Calculate the draughts on completion; Calculate where to load an additional heavy lift oJ 220 t on deck in order that the ship completes with a trim of 0.10 m by the stern. (F 5.144 m, A 4.865 m: 41.28 m foaP)

(a) iUi

TR|M BY (LcB-LcG) METHoD UslNG HYDRosTATlc DATA (AR Rev. 15/1ll02)

Answers Prepared by MCA OOW Nuri KAYACAN

Dec 2012

HYD ROSTATIC P ARTICUTARS

DRAUGHT
m

DISPL.
t

DISPL.
t

TPC
t

TPC
t

MCTC
t-m

MCTC
t-m

KMt
m

KB
m

LCB
foap

LCF
foap m 67.3s 67.46
67.57 67.68

SW

RD
7.00 6.90 6.80 6.70 6.60

1.025

FW SW FW RD 1.000 RD 1.025 RD 1.000


t4220
13996
13't71 23.13 22.57 22.50 22.43 22.36 22.29 22.23

SW RD
1.025

FW

m
8.34 8.35 8.36 8.37 8.38 8.39
8.41 8.43

RD

1.000

14576 14345

84.6 83.0 81.4 79.9


78.3

80 I

3.64

70.03 70.08
'70.12

23.06
22.99 22.92 22.85 22,78
22.'72

785
77.0 75.5 74.0 72.5

3.s8
3.53 3.48 3.43
3.3 8

t4t l5
l3 886 13657

t3548
13324

70.16 70.20 70.24 70.28


'70.32

67.79 67.90 68.00


68.

6.s0
6.40 6.30 6.20 5.10 6.00
5.90 5.80

13429
13201

13102

768
75.3

t2879
12658
12437 12217 11997

22.t7
22.t1 22.0s 2r.99
21.93 21.87 21.82 21.77 21.72

7t.0
69.6 68.3 67.0 65.7 64.4 63.2
62.1

J.JJ

2975 2748
2523

22.66 22.60
22.54 22.48 22.43 22.37 22.32 22.26 22.21 22.15

73.9
172.5 171.1
r

3.28 3.22

l0

846
8.49 8.52
8.55

70.35 70.38 70.42 70.46 70.50


'70.53

68.20

317
3.11

68.30
68.39 68.43 68.57

2297
2073
l 848

69.8

11778
I

168.5 167.3

3.06
3.01

1559

8.59
8.63

s.70
5.60 5.50 5.40

t62s
1402
I 180

11342 11124
10908
10691

66r
65.0
63.9

2.95

68.6s
68.73

61.0

8.67
8.71
8.'76

2.90
2.85 2.80 2.74 2.69 2.63 2.58 2.53 2.48 2.43 2.38

70.57 70.60
'70.64

2t.66
2t.61
21.56
21.51

60.0
58.9
5',7.9

68.80
68.88

0958

62.9 61.8 60.8 59.8 58.8 57.9

s.30
5.20 5.10 5.00

0737
05

10475

22.10 22.0s
22.00

8.81

70.68 70.72 70.75 70.79


'70.82

68.9s
69.02 69.09 69.76
69.23

l6

t0260
10045

56.9

8.86

t0296 t0076
9857

21.46

55.9 54.9 54.0


53. I

892
8.98 9.06

9830

21.9s
21.90 21.85 21.80 21.75 21.70 21.65 21.60 21.55 21.50 21.45 21.40 21.35 21.30 21.24

2t.4r
21.36 21.32 21.27 21.22

4.90
4.80
4.',70

9616
9403 9190 8978

9638 9420 9202 898s


8768

s6.9 s6.0
55. I

913
9.22 9.30 9.40 9.49 9.60
9.71

70.86 70.90
'70.93

69.29 69.35 69.42 69.48

52.2 51.3 50.5 49.6 48.7

4.60 4.50 4.40 4.30 4.20

8766 8554
8344
8133

21.t7
21.17 21.07 21.02 20.97 20.93 20.88 20.83 20.78 20.72 20.67

54.2

2.32
2.27

70.96 71.00 71.04 71.08

53.3 52.4
51 .5

69.s5
69.62 69.68 69.74
69.81

8552
8336

2.22
2.17 2.12

47.8
46.9

4.r0
4.00 3.90 3.80 3.70 3.60 3.50

8t2l
'7906 7692 74'78
'7265

7923 7713 7505 7296 7088 6880 6673

50.6

9.83 9.96

71.t2

r49.7 r48.7
147.8

46.0

207
2.01
1.96
1.91

71.\s
71.18 71.22
'71.2s

45.r
44.2
43.3

r0.ll
10.25
10.41

69.88 69.94

146.8 145.9 144.9

70.00 70.07
70.14

'7052

42.3

10.57 r0.76

1.86

71.29
71.33

6840

21.t9

413

181

THESE HYDROSTATIC PARTICULARS FIAVE BEEN DEVELOPED WITH THE VESSEL FLOATING ON EVEN KEEL.

Answers Prepared by MCA OOW Nuri KAYACAN

You might also like